Circulatory System/Heart/Vessel Flashcards

1
Q

Which one of the following is not a differential diagnosis for shoulder tip pain?

A. Pulmonary Embolism
B. Myocardial Infarction
C. Emphysema
D. Pneumothorax
E. Peptic Ulcer Disease

A

C. Emphysema

Shoulder-tip pain is an important clinical sign and can be caused by local musculoskeletal trauma or inflammation or referral. The following are the differential diagnoses of referred shoulder-tip pain:
- Pulmonary embolism
- Pneumothorax
- Myocardial infarction
- Perforation of peptic ulcer disease
- Diaphragmatic irritation

Emphysema is not a cause of shoulder-tip pain, unless a spontaneous pneumothorax occurs.

How well did you know this?
1
Not at all
2
3
4
5
Perfectly
2
Q

After percutaneous coronary intervention (PCI) in a patient with STEMI, which one of the following is the recommended period for anti platelet therapy?

A. Two weeks
B. Four weeks
C. Six weeks
D. 12 months
E. Lifelong

What are the common DAPTs?

A

In nearly all patients in whom intra-coronary drug-eluting stent is placed, dual antiplatelet therapy is indicated for at least 12 months. Dual therapy is with aspirin and any of the following:
- Clopidogrel
- Prasugrel
- Ticagrelor

Reference:
* RACGP - AFP - Dual antiplatelet therapy

How well did you know this?
1
Not at all
2
3
4
5
Perfectly
3
Q

Which one of the following is of predictive value for prognosis of a patient with systolic heart failure?

A. Jugular venous pressure.
B. Peripheral edema.
C. Shortness of breath.
D. Orthopnea.
E. Chest pain.

Which above is diagnostic and which is predictive?

A

A. JVP

Prognosis of systolic heart failure can be predicted by jugular venous pressure (JVP) and the third heart sound (S3). Increased JVP or the presence of S3 sound indicates poor prognosis.

Other options are of diagnostic, not predictive values.

How well did you know this?
1
Not at all
2
3
4
5
Perfectly
4
Q

Based on the culture results of a 42-year-old man with infective endocarditis, hemophilus ducreyi is found to be the causative organism. He is treated with a course of intravenous ceftriaxone. Which one of the following would be the most appropriate investigation in addition to treatment?

A. Colonoscopy.
B. Abdominal CT scan.
C. CT angiography.
D. Trans-thoracicechocardiography(TTE).
E. Trans-esophageal echocardiography (TEE).

A

C. CT angiography

Infective endocarditis (IE) can be uncommonly caused by HACEK group. HACEK group are oral gram-negative bacilli including Hemophilus, Aggregatibacter actinomycetmcomitans, Cardiobacterium hominis, Eikenella corrodens and Kingella kingae. HACEK organisms are most often associated with infective endocarditis, accounting for up to 10% of cases. They are also the most common cause of gram-negative endocarditis among persons who do not abuse intravenous drugs.

Generally, IE have several complications with embolic events being one of the most lethal ones.
A number of imaging studies have been used to identify infected aneurysms, including ultrasound, CT scan, MRI and digital subtraction angiography (DSA). Of these, CT angiography is the most useful one for diagnosing mycotic aneurysm. MRI angiography is the alternative when intravenous contrast is contraindicated.

Colonoscopy is considered in patients with Streptococcus bovis as the cause of their IE because Streptococcus bovis is found to be associated with increased risk of colon cancer. Abdominal CT scan is not useful for detection of microaneurysms.
Trans-thoracic or trans-esophageal echocardiography is not useful for detection of micoraneurysms.

How well did you know this?
1
Not at all
2
3
4
5
Perfectly
5
Q

You are called to see a 50-year-old Aboriginal woman with history of congestive heart failure, who has developed severe dyspnea and is in respiratory distress. On examination, she is pale and sweaty with a blood pressure of 110/75 mmHg and pulse rate of 120bpm. The respiratory rate is 26/min. Chest auscultation is significant for bilateral crackles. A chest X-ray shows bilateral whiteout of lungs from the lower lobes up to middle of the lung fields. An ECG shows no abnormality. Troponin level is normal on arrival and eight hours later. Which one of the following is the most likely diagnosis?

A. Acute right heart failure.
B. Acute pulmonary edema.
C. Pleural effusion.
D. Acute myocardial infarction.
E. Pulmonary embolism.

A

B. Acute pulmonary EDEMA - dyspnea, bibasilar coarse crackles, tachycardia, pallor and cold limbs due to hypoperfusion. The background history of CHF supports the diagnosis of pulmonary edema in this patient as the most likely diagnosis.

A. Acute RHF - ankle edema, raised JVP, hepatomegaly, shortenss of breath. With only lung congestion and no other findings that can be present in right heart failure, this diagnosis less likely.

C. Pleural effusion - progresses slowly and usually does not cause acute-onset symptoms. Even an acute pleural effusion takes hours to days to develop.

D. With acute MI - ECG changes and positive cardiac enzymes should have been present.

E. A massive pulmonary embolism (PE) can be the second likely diagnosis on the list differential diagnoses. PE presents with shortness of breath, tachypnea and tachycardia as the hallmark symptoms. Pleuretic chest pain or shoulder-tip pain may be other finding. ECG may be normal or show S1Q3T3 pattern (prominent S wave in lead I, Q wave and T wave inversion in lead III), sinus tachycardia, T wave inversion in leads V1 –V3 and right bundle branch block.

How well did you know this?
1
Not at all
2
3
4
5
Perfectly
6
Q

A 65-year-old woman presents with several episodes of acute lightheadedness, especially shortly after getting off the bed or a chair for the past 3 months. Her medical history is otherwise unremarkable. A table tilt test is arranged that is positive. Which one of the following would be the first-line management of this patient?

A. Oral fludrocortisones.
B. Oral hydrocortisone.
C. Intravenous fluids.
D. Increased salt and water intake.
E. Indomethacin.

A

D. Increased salt and water intake.

A positive table tilt test is highly suggestive of orthostatic (postural) hypotension
= Excessive fall in BP when an upright position is taken on. BP drop is >20mmHg in systolic pressure, 10mmHg in diastolic pressure or both. The condition may be acute or chronic.
= Most common acute causes include:
* Hypovolemia (e.g. blood lost)
* Drugs
* Prolonged bed rest
* Adrenal insufficiency
= Most common chronic causes:
* Age-related changes in blood pressure regulation
* Drugs
* Autonomic dysfunction
= Symptoms are related to diminished blood flow to central nervous system, especially the brain and include:
* Faintness
* Lightheadedness
* Confusion
* Blurred vision
* Syncope, falls or even seizures may be seen in severe cases

Too Long, Read Page 6/7

Conservative Management
should always be considered as the first-line management. These measures include increased sodium and water intake in the absence of heart failure or hypertension. This may expand intravascular volume and decreases the severity of symptoms. This approach carries the risk of heart failure, particularly in the elderly and patients with impaired myocardial function.

NOTE - development of dependent edema without heart failure is not a reason to stop the treatment.

Other conservative measures include:
* Patients requiring prolonged bed rest should be advised to sit up first on waking and exercise in bed when possible.
* Elderly patients should avoid prolonged standing. Sleeping with the head of the bed raised may relieve symptoms by promoting Na retention and reducing nocturnal diuresis.
* Patients should rise slowly from a recumbent or sitting position, consume adequate fluids, limit or avoid alcohol, and exercise regularly when possible, because modest-intensity exercise promotes overall vascular tone and reduces venous pooling.
* If the case is postprandial hypotension, it should be recommended that the size and carbohydrate content of meals be reduced. The patient must minimize alcohol intake and avoid sudden standing after meals.
* Waist-high fitted elastic hose may increase venous return, cardiac output, and BP after standing.

Medical Management
If conservative measures fails:
**Fludrocortisone **- being a potent mineralocorticoid, fludrocortisone exerts it effects through sodium retention, which results in volume expansion and relieving or decreasing the symptoms. It is only effective if sodium intake is adequate.

This drug may also improve the peripheral vasoconstrictor response to sympathetic stimulation. Supine hypertension, heart failure, and hypokalemia may occur; K supplements may be required.

Other drugs used include midodrine, NSAIDs, L-Dihydroxyphenylserine (a norepinephrine precursor), and propranolol or other beta blockers.

For this patient with no heart fialure, increased water and sodium intake will be the next best step in management.

How well did you know this?
1
Not at all
2
3
4
5
Perfectly
7
Q

John, 35 years old, presents to the emergency department with pain and swelling of his left thigh since this morning. Investigations establish the diagnosis of deep venous thrombosis for which he is started on heparin in hospital. He has diabetes and hypertension and his wife mentions that is very busy and distracted and always forgets to take the drugs he is prescribed for treatment of his hypertension and diabetes. Which one of the following options would be the most appropriate management for him after the course of heparin is completed?

A. No more treatment is needed.
B. Warfarin for 6 months.
C. Aspirin for 6 months.
D. Surgical intervention.
E. Caval filter.

DVT noncompliance

A

E. Caval Filter (Inferior Vena Cava (IVC) Filter)
As this patient is known be noncompliance with his medications he should have a inferior vena cava filter for prevention of PE.

Treatment of DVT starts with either unfractionated or low molecular weight heparin. Warfarin could be started at the same day (or within 48 hours). Heparin therapy should be continued for 5 days and stopped once INR is above 2 in two consecutive days.

NOTE - Since anticoagulation is contraindicated in the presence of a bleeding diathesis, the following tests should be performed prior to heparinization:
* APTT
* INR
* Platelet count
* Thrombophilia screen including: activated protein C resistance, fasting plasma homocysteine, prothrombin G20210A, antithrombin III, protein C, protein S, lupus anticoagulant, anticardiolipin antibody and lupus anticoagulant

Warfarin should be continued for at least 3 months or more depending on the patient’s risk of recurrent VTE.

The objectives of anticoagulation therapy are treating the current DVT and prevention of pulmonary embolism. Studies have shown that as many as 33% of patients may develop PE while receiving adequate anticoagulation therapy.

Cava filters are an alternative to systemic anticoagulation with warfarin (or heparin) in the following situation: DVT or PE in patients with contraindications to anticoagulation therapy; these patients include those with:
* Hemorrhagic stroke
* Recent neurosurgical procedure or other major surgery
* Major or multiple trauma
* Active internal bleeding (e.g. upper or lower gastrointestinal bleeding, hematuria, hemobilia)
* Intracranial neoplasm (either primary or metastatic)
* Bleeding diathesis (e.g. secondary thrombocytopenia, idiopathic thrombocytopenic purpura, hemophilia) Pregnancy
* Unsteady gate or tendency to fall (as seen in patients with previous stroke, Parkinson disease)
* Poor patient compliance with medications
…. long list not important.

Inferior Vena Cava (IVC) Filter

Due to the patient’s noncompliance with medication, an IVC filter should be used to prevent pulmonary embolism (PE).

DVT Treatment:
- Start with unfractionated or low molecular weight heparin.
- Begin warfarin on the same day or within 48 hours.
- Continue heparin for 5 days, stopping once INR is above 2 for two consecutive days.

Important Tests Before Starting Heparin (if anticoagulation is contraindicated):
- APTT
- INR
- Platelet count
- Thrombophilia screen (activated protein C resistance, fasting plasma homocysteine, prothrombin G20210A, antithrombin III, protein C, protein S, lupus anticoagulant, anticardiolipin antibody)

Warfarin Therapy:
- Continue for at least 3 months or longer, depending on the risk of recurrent VTE.

Goals of Anticoagulation Therapy:
- Treat current DVT
- Prevent PE (though 33% of patients may develop PE despite adequate therapy)

IVC Filters:
- An alternative to systemic anticoagulation in patients who cannot take anticoagulants.
- Use in patients with:
- Hemorrhagic stroke
- Recent neurosurgery or major surgery
- Major trauma
- Active internal bleeding (GI bleeding, hematuria, hemobilia)
- Intracranial tumors
- Bleeding disorders (thrombocytopenia, ITP, hemophilia)
- Pregnancy
- Unsteady gait or risk of falls (e.g., stroke, Parkinson’s disease)

This simplified approach ensures understanding of when to use IVC filters and the steps for DVT and PE prevention.

How well did you know this?
1
Not at all
2
3
4
5
Perfectly
8
Q

Which one of the following is not indicated in management of pulmonary edema?

A. Continuous positive airway pressure (CPAP).
B. Bilevel positive airway pressure (BiPAP).
C. Glyceryl trinitrate.
D. Oral forusemide.
E. Morphine.

A

D. ORAL Forusemide

Management of acute pulmonary oedema:
1. Oxygen 10-15 L/min by Hudson mask and reservoir bag.
2. Once the patient is stable continue oxygen 2-6 L/min by nasal cannula.
3. INTRAVENOUS forusemide - it is one of the most essential steps in treatment of pulmonary oedema by decreasing the volume overload. If taken orally, forusemide takes time to work and is not effective in treatment.
4. GTN – it reduces the preload; it is essential to titrate the dose to maintain systolic blood pressure above 100mmHg.
5. Morphine – by decreasing the sympathetic tone, it results in vasodilation and reduction of preload
6. CPAP and BiPAP – these non-invasive methods of ventilation are used to reduce alveolar and pulmonary edema by reducing the venous return and preload.

O2, reduce water, reduce preload/pain

How well did you know this?
1
Not at all
2
3
4
5
Perfectly
9
Q

A 76-year-old man comes to your clinic for a routine health check-up. He has blood pressure of 110/90 mmHg and a pulse rate of 92 bpm. He is on no medications except daily multivitamins. As a part of evaluation, an ECG is obtained which is shown in the following photograph. Which one the following is the next best step in management?

(ECG on page 12)

A. Aspirin.
B. Warfarin.
C. Reassurance.
D. Metoprolol.
E. Atropine.

A

C. Reassurance.

The ECG shows a sinus rhythm with increased PR interval characteristic of first-degree atrioventricular block. The condition can be caused by:

  • Age-related fibrosis and degeneration of AV node – the most common cause
  • Drugs: digoxin, beta blockers, calcium channel blockers
  • Increased vagal tone.

First-degree heart block is characterized by PR interval > 200 ms (0.2 s) on ECG, normal QRS complexes in terms of duration and spacing, and the presence of a P wave before each QRS.

This is quite common among the elderly due to age-related fibrotic changes of the cardiac conductive system.

Asymptomatic patients do not need treatment and must be reassured.

With symptoms (e.g., dizziness, shortness of breath, chest pain, etc.) atropine or pacemaker (if unresponsive to atropine) is considered.

How well did you know this?
1
Not at all
2
3
4
5
Perfectly
10
Q

A 79-year-old man collapses on the floor while waiting in the Emergency Department and becomes unresponsive. He has a blood pressure of 84/47 mmHg and rapid and barely perceptible pulse. Cardiopulmonary resuscitation is started immediately. The rhythm, obtained by defibrillator is shown in the accompanying photograph. Which one of the following is the next best step in management?

(ECG on page 13)

A. Continue CPR until the patient regains consciousness.
B. Cardioversion.
C. Amiodarone infusion.
D. Intravenous adrenaline.
E. Defibrillation.

A

B. Cardioversion.

The first step in management of a collapsed patient is management is calling for help (if possible) and starting CPR with chest compression and ventilation. Once the defibrillator is available obtain the cardiac rhythm with the pedals. The rhythm of this patient is characteristic of ventricular tachycardia (VT). In patients with unstable hemodynamic status, the next best step in management is synchronized cardioversion if the patient has a pulse, or defibrillation if no pulse is detected.

Hemodynamic instability is manifested as:
* Chest pain
* Dyspnea
* Hypotension
* Perfusion-related confusion
* Collapse and/or unresponsiveness

Option A: Continuing the CPR without defibrillation is unlikely to help in this condition.

Option C: Amiodarone and other anti-arrhythmic drugs used for VT are indicated if the patient is hemodynamically stable.

Option D: IV adrenaline is the first step in management, in conjunction with chest compression and ventilation, in patients with asystole, or ventricular fibrillation after two attempts of DC shock fail to convert the rhythm.

Option E: Defibrillation is the option when there is ventricular fibrillation or if the patient has pulseless VT.

How well did you know this?
1
Not at all
2
3
4
5
Perfectly
11
Q

Which one of the following is NOT a contraindication to thrombolytics use in a patient with ST elevation myocardial infarction?

A. Ischemic stroke in the past 3 months.
B. Hemorrhagic stroke in the past 10 years.
C. Heavy menstrual bleeding.
D. Coagulation disorders.
E. Gastrointestinal bleeding within the past 4 weeks.

A

C. Heavy menstrual bleeding.

Absolute contraindications for fibrinolytic use in STEMI include the following:
* Prior intracranial hemorrhage (ICH)
* Known structural cerebral vascular lesion
* Known malignant intracranial neoplasm
* Ischemic stroke within 3 months
* Suspected aortic dissection
* Active bleeding or bleeding diathesis (excluding menses)
* Significant closed head trauma or facial trauma within 3 months Intracranial or intraspinal surgery within 2 months
* Severe uncontrolled hypertension (unresponsive to emergency therapy)
* For streptokinase, prior treatment within the previous 6 months

Relative contraindications for fibrinolytic use in STEMI include the following:
* History of chronic, severe, poorly controlled hypertension
* Significant hypertension on presentation (systolic blood pressure >180 mm Hg or diastolic blood pressure >110 mm Hg
* Traumatic or prolonged (>10 minutes) cardiopulmonary resuscitation (CPR) or major surgery less than 3 weeks previously
* History of prior ischemic stroke not within the last 3 months
* Dementia
* Recent (within 2-4 weeks) internal bleeding
* Noncompressible vascular punctures
* Pregnancy
* Active peptic ulcer
* Current use of an anticoagulant (e.g., warfarin) that has produced an elevated INR higher than 1.7 or a PT longer than 15 seconds

Of the given options, only heavy menstrul bleeding is not an absolute contraindication to thrombolytic therapy.

How well did you know this?
1
Not at all
2
3
4
5
Perfectly
12
Q

A 67-year-old man presents for assessment after he experienced an episode of vision loss of his right eye lasting 30 minutes 24 hours ago. The condition is resolved now. On examination, the visual acuity of the left and right eyes are 6/60 and 6/36, respectively. A systolic murmur is noted over the sternum. No carotid bruit is heard. Which one of the following is most likely to establish the cause of his condition?

A. CT scan of the brain.
B. Electroencephalography (EEG).
C. MRI of the brain.
D. Doppler ultrasound of carotid arteries.
E. Echocardiography.

A

D. Doppler ultrasound of carotid arteries

In adults, transient visual loss is a frequently encountered complaint that in most cases has an identifiable cause. The loss of vision may be monocular or bilateral and may last from seconds to hours. Episodes are usually ischemic in origin.

Ischemic causes of transient visual loss include:
-Giant cell arteritis
-Cerebrovascular ischemia
-Retinal arteriolar emboli
-Amaurosis fugax syndrome

Transient visual loss can be a symptom of a serious vision-threatening or even life-threatening condition, requiring urgent investigation and treatment, or it may have a more benign origin (e.g., migraine). Transient visual loss in children is less common than in adults and is more likely to have a benign origin. Causes of transient visual loss in children include migraine and epileptic seizure.
Embolic occlusions of the arteries supplying the eye are a common cause of transient visual loss in adults. Emboli causing circulatory compromise may originate from the heart or the carotid arteries. Embolic events are usually isolated; therefore, frequent episodes of visual loss are less likely to be caused by emboli.
In contrast to transient ischemic attacks (TIAs) involving the cerebral hemispheres, retinal ischemia is more commonly associated with emboli originating from carotid stenosis rather than the heart.

Diagnostic evaluation of transient visual loss (TVL):
The overlap in clinical presentations and grave prognosis of some potential diagnoses necessitates that some diagnostic testing is performed in most patients:
Ophthalmologic evaluation - A detailed fundoscopic evaluation is an important part of the evaluation of patients with transient visual loss. Ophthalmology referral is required for all patients with suspected giant cell arteritis, retinal vein disease, and ocular causes of visual loss.
Erythrocyte sedimentation rate (ESR) and C-reactive protein - All older patients (>50 years) with transient monocular or binocular vision loss should have an ESR and C-reactive protein to exclude giant cell arteritis (GCA). If these are elevated, or if the history is very suggestive, patients should proceed to a confirmatory temporal artery biopsy. Treatment with predniso(lo)ne should be started empirically.
Carotid imaging - Carotid Duplex Doppler ultrasound, magnetic resonance angiography (MRA), or computed tomographic angiography (CTA) should be ordered in all older patients (>50 years) and in younger patients with vascular risk factors (diabetes, hypertension, hyperlipidemia), who have experienced transient mono-ocular visual loss (TMVL).
Duplex ultrasound is the preferred initial modality because it is noninvasive, inexpensive and readily available with acceptable sensitivity and specificity.
Cardiac evaluation - Once GCA and carotid disease have been excluded, an evaluation to see if a cardiogenic source of embolism exists should follow in all older patients (and younger patients with risk factors), who have had TMVL. This is also indicated in patients with transient binocular visual loss (TBVL) due to posterior circulation ischemia. Testing may include Holter monitoring and echocardiography. A baseline electrocardiogram (ECG) should also be included in the evaluation of these patients because cardiac morbidity and mortality in patients with TMVL and central retinal artery occlusion is significant.
Brain MRI - Older patients with binocular visual symptoms (TBVL) with accompanying symptoms suggestive of vertebrobasilar ischemia should have a brain MRI.
Electroencephalography (EEG) - EEG is not a routine test for TVL, but should be performed in a patient with TBVL whose symptoms suggest possible seizure. EEG monitoring may increase the diagnostic yield, especially in patients with frequently recurring symptoms.
Hypercoagulable testing - When brain or ocular ischemia is the suspected cause of TVL, hypercoagulable testing should be performed in individuals who have suggestive histories (prior thrombosis, miscarriage, or family history), as well as in individuals with probable ischemia and otherwise negative workup. A full blood exam should also be obtained to screen for conditions such as polycythemia vera and essential thrombocythemia.
Of the options, the most important initial test to consider is Duplex Doppler ultrasonography of carotid arteries since most visual losses are due to carotid artery stenoses. Absence of carotid bruit does not exclude the possibility of carotid stenosis as the most likely cause of retinal ischemia in this patient.

How well did you know this?
1
Not at all
2
3
4
5
Perfectly
13
Q

A 70-year-old man presents to the Emergency Department with complaint of chest pain starting 15 minutes ago, central in location, and dull and aching in nature. He is given aspirin, sublingual glyceryl trinitrate, and oral antacid, and is put on supplemental oxygen by nasal cannula. These measures ameliorate the pain to a significant extent.
On examination, his blood pressure is 140/90mmHg, pulse 110 bpm, and respiratory rate 20 breaths per minute. A 12-lead ECG strip reveals no abnormality. He mentions that he has had these pains every time he exceeded a certain amount of physical activity, and that each time the pain subsides with rest or sublingual glyceryl trinitrate. He rates this current episode no more than the previous ones. Which one of the following is the most appropriate next step in management?

A. Reassure and discharge him home.
B. Book for an outpatient echocardiography.
C. Admit him to coronary care unit (CCU), measure cardiac enzymes and repeat the ECG.
D. Refer him for a stress ECG and echocardiography.
E. Refer him to a gastroenterologist.

A

D. Refer him for a stress ECG and echocardiography.

Central chest pain described as heaviness, dull and aching that may or may not radiate to the jaw, left arm, or epigastrium is more likely to be ischemic in nature. The duration of pain (< 20 minutes), being brought on by a predictable amount of exertion and relieved by rest or nitrates is characteristic of stable angina, a condition in which an increased oxygen demand of the heart, induced by activity, leads to ischemia because the stenotic coronary arteries cannot keep up with increased need for oxygen. On the other hand, unstable angina is defined as any new ischemic chest pain, or one with deviation from the typical pattern of previous pains, in terms of either duration, intensity, frequency, or decreased amount of exertion required for its reproduction.

The characteristics of the pain (duration, reversibility, and response to rest and nitrates), in addition to a normal ECG establishes the diagnosis of stable angina in this patient. In approaching to such patients, the next best step in management is urgently performing a stress test (either conventional treadmill, chemical, or nucleic) to establish the coronary artery stenosis if the ECG or diagnosis is equivocal (as is in this patient). A positive stress test (reproduction of the chest pain, ST segment depression>2mm or a drop in blood pressure>10mmHg) is then followed by angiography for further evaluation and treatment with either ballooning with or without stent placing or coronary artery bypass grafting surgery.

Option A: Reassuring and discharging the patient is not an appropriate action before the patient has been fully assessed.

Option B: Echocardiography may be a part of plan now or later on in outpatient setting, but is not the most appropriate management now.

Option C: Admission to CCU, cardiac enzymes and follow-up ECGs were indicated if the patient had any changes in the intensity of the pain or its duration, unresponsiveness to rest and nitrates, or aggravating or relieving factors.

Option E: Referring the patient to gastroenterologist is not correct because the pain is typical for cardiac ischemia.

How well did you know this?
1
Not at all
2
3
4
5
Perfectly
14
Q

A 50-year-old man presents to the Emergency Department with chest pain felt behind the sternum radiating to his jaw. A 12-lead ECG strip is obtained and is as the following photograph. You give him aspirin and sublingual nitroglycerine and start him on supplemental oxygen by nasal cannula. A troponin level is ordered which comes back negative. Which one of the following is the next best step in management of this patient?

(ECG on Page 16)

A. Repeat troponin in 8 hours.
B. Immediate reperfusion therapy.
C. Repeat the ECG in 6 hours.
D. Start him on beta blockers.
E. Serial ECGs.

A

B. Immediate reperfusion therapy.

With ST elevation in leads II, III, and aVF on the given ECG, this patient has sustained an inferior ST-elevation myocardial infarction (STEMI) for which the next best step in management is either immediate percutaneous coronary intervention (PCI) as the preferred option, or thrombolytic therapy if not contraindicated.

Option A: Even with negative troponin, ST elevation of more than 1mm in two or more contiguous leads and chest pain makes the diagnosis certain. In fact with chest pain and ECG changes, no troponin level was required to guide the management, and reperfusion therapy should have been performed even without waiting for the results.

Option C: This patient has acute inferior MI and should be treated immediately. Waiting for 6 hours to obtain an ECG is definitely an incorrect answer.

Option D and E: Serial ECGs are indicated as well to further assess the possible evolution of the myocardial infarction. Beta blockers are effective in reducing the mortality, but neither ECG, nor beta blockers takes precedence over PCI.

How well did you know this?
1
Not at all
2
3
4
5
Perfectly
15
Q

A 60-year-old man presents with complaints of increasing tiredness and abdominal distention for the past four months. His past medical history is remarkable for smoking 20 cigarettes a day for the past 20 years. On examination, there is bilateral ankle edema and ascites. The liver is palpated 3cm below the costal margin. His jugular pulse is noted to drop on expiration and rise on inspiration. Which one of the following is the most likely diagnosis?

A. Cardiac tamponade.
B. Budd- Chiari syndrome.
C. Superior vena cava obstruction.
D. Constrictive pericarditis due to TB in the past.
E. Hepatic cirrhosis.

What is the special sign seen here? Typically seen in what conditions?

A

D. Constrictive pericarditis due to TB in the past.

Drop of jugular pulse on inspiration and its rising during expiration is a normal physiologic response. Dropped jugular venous pressure (JVP) during expiration and its rise on inspiration is a pathological sign called Kussmal sign. Kussmal sign is seen in restrictive cardiomyopathy, constrictive pericarditis, and cardiac tamponade. Of the options, only constrictive pericarditis due to TB infection can present with Kussmal sign.

Option C: Superior vena cava (SVC) obstruction leads to edema of the face not ankle edema. On the other hand, although the JVP is raised in SVC obstruction, there is no pulsation of the jugular vein.

Option B: Budd-Chiari syndrome is associated with thrombus formation in the hepatic vein, leading to portal hypertension. It may present with fatigue, right upper quadrant pain, mild jaundice, and hepatosplenomegaly; however, JVP remains normal, without pulsation.

Option E: In hepatic cirrhosis , the liver is usually shrinked and not enlarged. Although fatigue, edema and ascites are commn findign, the Kussmal sign is not a feature.

Cardiac Tamponade p/w Beck’s Triad (hypotension, JV distension, muffled)

How well did you know this?
1
Not at all
2
3
4
5
Perfectly
16
Q

A 30-year-old man presents to the emergency department with chest pain that has started this morning and worsened over time. He mentions that deep breathing increases the pain intensity. On examination, pleuretic chest pain and a temperature of 38°C is noted. The BP is 140/85mmHg and the pulse 100bpm. He takes shallow breaths in a rate of 20/min. The rest of physical examination is inconclusive. A 12- lead ECG is obtained and is shown. Which one of the following is the most likely diagnosis?

(ECG on Page 18)

A. Myocardial ischemia.
B. Pulmonary embolism.
C. Infectious endocarditis.
D. Acute myocardial infarction.
E. Pericarditis.

A

E. Pericarditis.

The clinical picture of pleuretic chest pain (chest pain worsened with breathing), mildly elevated respiratory rate and a borderline pulse rate can be either to pericarditis or pulmonary embolism, but diffuse ST elevation in pericordial and limb leads favors pericarditis as the most likely diagnosis.

Option A: Although myocardial ischemia causes chest pain, the nature of the pain and the pattern of ST segment elevation makes this diagnosis less likely.

Option B: Pulmonary embolism can present similarly; however, the ECG changes are not consistent with this diagnosis.

Option C: Infectious endocarditis presents with a murmur and fever. The ST changes in the ECG are characterisitc for pericarditis. Infectious endocarditis does not cause such an ECG abnormality.

Option D: Acute myocardial infarction (MI) causes ST elevation in specific lead groups, depending on the coronary artery involved and its territory. The pleuretic nature of the chest pain, on the other hand, is against MI as a likely diagnosis.

How well did you know this?
1
Not at all
2
3
4
5
Perfectly
17
Q

A 70-year-old man in brought to the Emergency Department because of light-headedness for the past 4 hours. On examination, he is found to have bradycardia with an irregular pulse of 45 bpm and a blood pressure of 85/60 mmHg. Atropine is used as the treatment of symptomatic bradycardia but the pulse rate remains the same and the lightheadedness persists. An ECG strip is obtained which is shown in the following photograph. Which one of the following is the next best step in management?

A. Metoprolol.
B. Dopamine.
C. Intravenous pacemaker.
D. Permanent pacemaker.
E. Adrenaline.

A

C. Intravenous pacemaker.

The rather constant PR intervals, and ‘p’ waves that are not followed by a QRS complex seen on ECG is characteristic of Mobitz II AV block.

Symptomatic AV block should be initially be treated with temporary pacing. Percutaneous or intravenous pacemakers are often available in the emergency department and can be applied. If not, atropine is used instead.

In the following situations temporary pacemakers are the most appropriate option for the initial management:
* History of asystole
* Mobitz II AV block
* Complete heart block
* Ventricular standstill>3 seconds

Option A: Metoprolol is definitely the wrong option because using it in this situation can dramatically worsen the condition.

Option B: Dopamine is a drug used for increasing cardiac contractility and vascular tone and is not used in treatment of symptomatic bradycardias.

Option D: Permanent pacemakers are definitive treatment of such blocks and are considered after initial temporary pacing.

Option E: Adrenaline infusion is sometimes indicated to maintain an adequate heart rate after atropine, while waiting for pacemaker insertion.

See Page 19 for topic review on Second Degree (Mobitz) Heart Block.

Mobitz II AV Block: P and QRS doing its own thing

How well did you know this?
1
Not at all
2
3
4
5
Perfectly
18
Q

A 47-year-old man has developed central chest pain one hour ago. On examination in the Emergency Department he is sweating profusely, has a BP of 90/60 mmHg and a pulse rate of 50bpm. An ECG shows 2-mm ST elevation in leads II, III, and aVF. Which one of the following is the most common cause of death in pre-hospital setting in this condition?

A. Ventricular tachycardia.
B. Ventricular fibrillation.
C. Bradycardia.
D. Asystole.
E. Hypotension.

A

B. Ventricular fibrillation.

The clinical and ECG findings are characteristic of inferior ST-elevation myocardial infarction (STEMI).

Of all patients experiencing acute myocardial infarction (MI), usually in the form of ST-elevation MI, 25–35% will die of sudden cardiac death (SCD) before receiving medical attention, most often from ventricular fibrillation; however, ventricular tachycardia is the most common arrhythmia early in the course of MI.

Patients suspected of having STEMI should be connected to defibrillator on the way to the hospital. Most ventricular fibrillations occur in the first 24 hour post-MI, with a half occurring within the first hour.

How well did you know this?
1
Not at all
2
3
4
5
Perfectly
19
Q

A 59-year-old man in brought to the emergency department of a tertiary hospital with compressing chest pain that has started 30 minutes ago.The pain is central in location and radiates to his jaw and left arm. A 12-lead ECG is obtained and is as follows. Which one of the following is the most appropriate management of this patient?

(ECG on Page 22)

A. Fibrinolytic therapy.
B. Morphine.
C. Aspirin.
D. Cardiac catheterization.
E. Rescue angioplasty.

A

D. Cardiac catheterization.

The history is highly suggestive of angina pectoris. On ECG, there is ST elevation in leads I, aVl, V5 and V6 (lateral aspect of the heart) and ST elevation in V3 and V4 indicative of involvement of the anterior wall. The ECG is characteristic of an anterolateral ST-elevation myocardial infarction (STEMI).

The most appropriate management of patients with STEMI is emergency reperfusion therapy either by fibrinolytics or percutaneous coronary intervention (PCI) if the presentation is within 12 hours after the onset of chest pain. Since the patient is in a tertiary hospital, PCI is the preferred method of reperfusion therapy.

Provided that proper facilities and an experienced cardiac interventionist is available a PCI could be performed with 90 minutes after presentation. Cardiac catheterization is a general term that includes angioplasty, PCI, and balloon angioplasty.

Option A: Fibrinolytics are the preferred method if PCI cannot be performed and the patient has no major risk of bleeding.

Option B and C: Aspirin and morphine have to be administered for every patient with acute coronary syndrome as important steps but they are the primary steps and can be given while the patient is arranged to be transferred to catheterization laboratory. Aspirin could be the correct answer if the question asked about the next step.

Option E: Rescue angioplasty is a term used to describe emergency angiographic coronary intervention after fibrinolytics fail to control the ischemia.

How well did you know this?
1
Not at all
2
3
4
5
Perfectly
20
Q

Steven is a 65-year-old man, who is a known case of congestive heart failure. He had been stable on enalapril, metoprolol and digoxin until three weeks ago when his wife passed away from breast cancer. Since then, he stopped taking his medication. Today, he is brought to the emergency department by his son, with complaints of shortness of breath, night coughs and ankle edema. On examination, he has a blood pressure of 90/75mmHg, heart rate of 68 bpm, and respiratory rate of 26 breaths per minute. On auscultation, an S3 gallop is noted, but there is no crackle. You decide to start him on medication again. Which one of the following would be the best option to start with?

A. Start him on enalapril.
B. Start him on metoprolol.
C. Start him on digoxin.
D. Start him on enalapril, digoxin and metoprolol at the same time.
E. Start him on enalapril and metoprolol.

A

A. Start him on enalapril.
The case describes a patient, with congestive cardiac failure, who has been under control with three medications, but his heart condition has been decompensated due to drug withdrawal.

At this moment, and based on physical findings, the patient has not pulmonary edema (no comments on basal crackles).

Angiotensin converting enzyme (ACE) inhibitors (e.g. enalapril) improve prognosis in all patients in all grades of heart failure and should be used as initial therapy in all patients. Angiotensin II receptor blockers (ARBs) such as losartan are used when ACE inhibitors cannot be tolerated .

Diuretics are added to ACE inhibitors to help control congestive symptoms and signs.

Beta blockers should only be started when the patient is stable and euvolemic. As this patient has S3 gallop and ankle edema (signs of hypervolemia), any options suggesting beta blockers (e.g. metoprolol) as initial treatment would be incorrect.

Digoxin is used in patients with heart failure if:
* Heart failure is caused by atrial fibrillation (often in conjuction with beta blockers if the patient is euvolemic)
* In patients with heart failure, who have sinus rhythm, but medications such as ACE inhibitors, diuretics have not adequately controlled their symptoms
* In patients with significantly decreased ejection fraction (EF) (<35%)

How well did you know this?
1
Not at all
2
3
4
5
Perfectly
21
Q

Which one of the following drug groups is the mainstay of therapy in diastolic heart failure?

A. ACE inhibitors and beta blockers.
B. Beta blockers and calcium channel blockers.
C. Calcium channel blockers and diuretics.
D. Angiotensin receptor blockers and beta blockers.
E. Beta blockers and diuretics.

A

B. Beta blockers and calcium channel blockers.

Diastolic dysfunction is defined as decreased compliance of the left ventricle during the diastole, resulting in decreased cardiac output. A hypertrophied left ventricle due to chronic hypertension remains the leading cause of diastolic dysfunction. The disease is most common among the elderly women. Other causes of diastolic dysfunction include restrictive cardiomyopathy and constrictive pericarditis. Pericardial tamponade causes acute diastolic dysfunction. Ejection fraction essentially remains unaffected or even increases.

Mainstay of therapy in the diastolic dysfunction, also termed as ‘heart failure with preserved systolic function (HFPSF)’ largely depends on the underlying cause. Mainstay of pharmacotherapy is with cardiac selective beta blockers (e.g. atenolol, carvedilol, metoprolol) and/or calcium channel blockers. Beta blockers, by slowing the heart rate, give ventricles more time to fill and enhance cardiac output. Calcium channel blockers serve the same purpose and can be used as second-line treatment or added to beta blockers later in the course of treatment.

The following drugs should not be used in treatment of diastolic dysfunction:
* Diuretics – they may decrease the already prone-to-decrease cardiac output. They are indicated if there is pulmonary congestion and edema.
* Vasodilators (e.g. nitrates) – they decrease the venous return and decrease cardiac output.
* Arterial vasodilators (hydralazine) – they may cause dynamic obstruction against the left ventricle outflow.
* Digoxin and other inotropic drugs – there is no role for these drugs as the ejection fraction is normal or even elevated. Digoxin may be considered for co-existing atrial fibrillation (but not as the first-line therapy).

Option A: No clear evidence exists to support ACE inhibitor therapy for diastolic heart failure. There has been no direct improvement in overall morbidity and mortality in patients with diastolic heart failure associated with ACE inhibitors; however, ACE inhibitors may have an important role in the treatment of the diseases underlying diastolic heart failure. Moreover, patients with diastolic heart failure frequently have comorbidities such as renal insufficiency and care should be taken when using ACE inhibitors, as there is the risk of renal function deterioration.

Option C: Calcium channel blockers are useful for treatment of diastolic heart failure, but diuretics decrease the cadiac output and should be avoided, unless there is volume overload.

Option D: Beta blokcers are mainstay of therapy in patients with diastolic heart failure, but there is no clear evidence from randomized clinical trials that treatment with angiotensin receptor blockers directly improves overall morbidity or mortality, or diastolic function in patients with diastolic heart failure.

Option E: Diuretics should not be used in patients with diastolic dysfunction, unless in the presence of volume overload and congestion.

How well did you know this?
1
Not at all
2
3
4
5
Perfectly
22
Q

An 17-year-old girl sustained sudden loss of consciousness and collapsed while playing basketball at school. She is now in the emergency department. She mentions that her father died suddenly at the age of 37 years. Which one of the following investigations is most likely to establish the diagnosis of the underlying cause of her collapse?

A. ECG.
B. Cardiac biomarkers.
C. Angiography.
D. Chest X-ray.
E. Echocardiography.

A

E. Echocardiography.

Sudden loss of consciousness in a young person while on exertion is most likely due to either an arrhythmia or hypertrophic obstructive cardiomyopathy (HOCM). With a positive family history for sudden death at young age, HOCM would be considered as the most likely diagnosis.

CXR, ECG and electrocardiography are all part of the work-up for this patient, but among them, echocardiography is more likely to establish the diagnosis of HOCM. Echocardiography is the best tool for detecting structural cardiac abnormalities (e.g. valvular lesion, cardiomyopathies).

How well did you know this?
1
Not at all
2
3
4
5
Perfectly
23
Q

Patients with a variety of cardiac conditions may require special care in pregnancy. Which one of the following cardiac lesions is mots likely to cause problems during pregnancy?

A. Mitral stenosis.
B. Tricuspid regurgitation.
C. Mitral regurgitation.
D. Aortic regurgitation.
E. Ventricular septal defect.

A

A. Mitral stenosis.

Among given options, mitral stenosis (MS) is more likely to complicate a pregnancy. Plasma volume is increased by 50% during pregnancy leading to more pronounced left atrial congestion and backflow of blood to the lungs resulting in pulmonary congestion and its typical presentation (fatigue, dyspnea, orthopnea, paroxysmal nocturnal dyspnea, pulmonary oedema, etc).

Other mentioned lesions are well tolerated during pregnancy owing to the fact that decreased peripheral vascular resistance causes accumulation of blood in the periphery. This eliminates a part of the work load the heart should have undergone if all that blood returned to the heart for recirculation.

How well did you know this?
1
Not at all
2
3
4
5
Perfectly
24
Q

A 70-year-old man, farmer by profession, presents to the Emergency Department after he experienced chest pain of 20 minutes duration following physical activity. When the pain started, he took 3 puffs of glyceryl trinitrate but there was no relief. In the Emergency Department though, the pain alleviated with oxygen. On further questioning, you realize that he is on aspirin, metoprolol and sublingual glyceryl trinitrate spray on an ‘as needed’ basis. During the past 6 months, he has had only 3 episodes of chest pain subsided just by rest. He saw his GP 6 months ago when he received his last prescription. Which one of the following is the most likely explanation of failure of glyceryl trinitrate to relieve his symptoms?

A. Onset of unstable angina.
B. Nitrate tolerance.
C. Reduced drug potency.
D. Poor absorption through the mouth.
E. Onset of myocardial infarction.

A

C. Reduced drug potency.

The given scenario is more likely due to expiration of the glyceryl trinitrate. It is important to remember to remind the patients to update their GTN sprays or pearls. Expired GTN has reduced potency and effect on relieving the ischemic pain. Since this patient has refilled his prescription 6 months ago, it is likely that the drug is past its expiry date and lost the desired potency.

Option B: Nitrate tolerance is a possibility and concern in patients who take GTN or other nitrates on a regular basis. After some time, resistance to nitrates develops and the ability of pain control diminishes. In this patient, however, tolerance is very unlikely, as he is not been a regular user of nitrates and has used them occasionally on an ‘as-needed’ basis.

Option A and E: In patients with unstable angina or myocardial infarction, the pain may not respond to nitrates. However, this patient is not likely to have these because it is unlikely for unstable angina or myocardial infarction to respond to oxygen but not to nitrates.

Option D: GTN adequately and effectively absorbs through the mouth.

How well did you know this?
1
Not at all
2
3
4
5
Perfectly
25
Q

A 25-year-old woman comes to your practice with complaints of ‘skipped heart beats’. On examination, she has a blood pressure of 110/85 mmHg and pulse of 88 bpm. Heart auscultation is remarkable for a mid-systolic click followed by a late systolic murmur. Echocardiography shows mitral valve prolapse (MVP). Which one of the following is correct about her condition?

A. MVP is more common in men.
B. Prophylaxis against infectious endocarditis is recommended.
C. MVP is present in up to 10% of population.
D. Risk of pulmonary embolism is high.
E. Ventricular arrhythmias do not occur.

A

C. MVP is present in up to 10% of population.

Mitral valve prolapse is seen in up to 10% of population and is more common in women. Although it is asymptomatic most of the time, it can present with palpitation and shortness of breath, or rarely even chest pain or more severe complications.

Option A: MVP is more common in women than men.

Option B: Antibiotic prophylaxis against endocarditis is no longer recommended for valvular lesions in the 2007 AHA guideline. Indications for prophylaxis antibiotic therapy include bicuspid aortic valve, acquired aortic or mitral valve disease (including mitral valve prolapse with regurgitation), and hypertrophic cardiomyopathy with latent or resting obstruction.

Option D: If MVP leads to atrial enlargement and blood stagnation, there is an increased risk of systemic thromboembolism; however, risk of pulmonary embolism is extremely low.

Option E: Although very rarely, supraventricular and ventricular tachyarrhythmias are found in patients with MVP.

How well did you know this?
1
Not at all
2
3
4
5
Perfectly
26
Q

A patient with aortic stenosis has undergone cardiac catheterization. You are told that the aortic systolic gradient is 55 mmHg. Which one of the following additional piece of information is necessary to determine the significance of the gradient?

A. Presence of coronary artery disease.
B. Left ventricular ejection fraction.
C. Left ventricular end-diastolic pressure.
D. Cardiac output.
E. Left atrial pressure.

A

D. Cardiac output.

An aortic systolic gradient of 55 mmHg means that during systole, the pressure in the aorta is 55 mmHg lower than the pressure in the left ventricle. A normal gradient is less than 10 mmHg.

With aortic stenosis, the cardiac output is diminished because the stenotic valve impedes blood outflow from the left ventricle. Therefore, cardiac output would be determinant as to whether immediate surgical valve replacement should be considered.

It should be born in mind that even in the presence of a normal cardiac output, a gradient more than 50 mmHg needs surgical replacement of the aortic valve.

AMC Handbok - Page 435

How well did you know this?
1
Not at all
2
3
4
5
Perfectly
27
Q

A 39-year-old woman presents with dizziness after she left the gym. An ECG is obtained and is as the accompanying photograph. Which one of the following is the next best step in management?

A. Reassurance.
B. Intravenous atropine.
C. Transcutaneous pacemaker.
D. Intravenous pacemaker.
E. Adrenaline.

A

B. Intravenous atropine.

The rhythm strip shows sinus bradycardia with a rate of 50 bpm. Asymptomatic bradycardias are usually left alone. If symptomatic, symptoms can include:
* Pre-syncope or syncope (faint)
* Shortness of breath
* Lightheadedness (dizziness)
* Chest pain

Every patient with symptomatic bradycardias should be managed as follows:

Medical management:
* Atropine (IV 500-600mcg – repeat every 3-5 minutes up to 3mg) – the best initial management
* Adrenaline (IV infusion 2-10 mcg/min) to maintain satisfactory heart rate – the second-line medical management

Consider pacemaker insertion for the following patients:
Patients who do not respond adequately to medical therapy
Patients at risk of asystole:
* History of recent asystole Mobitz II AV block on ECG
* 3rd degree AV block on ECG Pulse < 40bpm
* Ventricular standstill > 3s

The initial pacing is with percutaneous pacing in the emergency department. Intravenous pacemaker insertion needs specialist referral. Permanent pacing is then followed electively if needed.

How well did you know this?
1
Not at all
2
3
4
5
Perfectly
28
Q

A 70-year-old man presents to the Emergency Department with chest discomfort, shortness of breath and pre-syncope. An ECG is obtained and is as follows. Which one of the following is the most appropriate immediate management of this patient?

A. Atropine.
B. Adrenaline.
C. Transcutaneous pacemaker.
D. Synchronized DC cardioversion.
E. Intravenous fluids.

A

C. Transcutaneous pacemaker.

The ECG is suggestive of third-degree (complete) heart block. Although atropine is used as first-line therapy for symptomatic bradycardias, in complete heart block emergency rescue transcutaneous pacemaker is the management of choice. Atropine should be given if emergency pacing cannot be done immediately.

ECG features:
* Severe bradycardia due to absence of AV conduction.
* The ECG demonstrates complete AV dissociation, with independent atrial and ventricular rates.

How well did you know this?
1
Not at all
2
3
4
5
Perfectly
29
Q

A 4-year-old boy is brought to the Emergency Department after he told her mother ‘my head is light and I feel like falling down’. On examination, his blood pressure is 80/50 mmHg and he has a regular pulse rate of 60 bpm. An ECG is obtained which is shown in the following photograph. Which one of the following is the next best step in management?

A. Give him adenosine.
B. Give him amiodarone.
C. Give him atropine.
D. Immerse his face in cold water and massage his carotid sinus.
E. Cardioversion with DC shock.

A

C. Give him atropine.

They ECG shows a sinus rhythm at rate of 60bpm, rather** wide QRS complexes** and the presence of typical ‘delta waves’ characteristic of a pre- exciting syndrome (e.g., Wolff-Parkinson-White). Although a heart rate of 60 bpm is considered normal in adults, it definitely signifies bradycardia in a 4-year-old child (normal 80-120bpm). Since this child has symptomatic bradycardia (light-headedness), he should be treated initially with atropine.

The normal range of heart rate for different age group is as follow:

< 1 y/o: 100-160BPM
1-2 y/o: 90-150BPM
2-5 y/o: 80-140BPM
6-12 y/o: 70-120BPM
>12 y/o: 60-100BPM

Option A and D: Adenosine or immersion of the face in cold water and carotid sinus massage are treatment options for supraventricular tachycardias, which is not the case here.

Option B: Amiodarone is used for ventricular tachyarrhythmias; this child does not have tachycardia.

Option E: Cardioversion is the treatment of choice for tachyarrhythmias that have resulted in hemodynamic instability, namely chest pain, shortness of breath, hypotension or hypoperfusion-related confusional state.

How well did you know this?
1
Not at all
2
3
4
5
Perfectly
30
Q

A 28-year-old man has had several episodes of palpitations associated with mild dizziness and sweating over the past year. These attacks occur usually after exercise and last between 10 and 20 minutes, and often stop after he immerse his face in cold water. He now presents with an attack that has been going on for 2 hours. His ECG is shown in the following picture. Which one of the following medications is most likely to be effective?

A. Digoxin.
B. Quinidine.
C. Flecainide.
D. Verapamil.
E. Propranolol.

A

D. Verapamil.

The rhythm strip shows paroxysmal supraventricular tachycardia (PSVT) at a rate of almost 180 bpm. The mechanism of PSVTs is either a re-entry circuit or automatous focus involving the atria. The best initial step in management of PSVTs is performing maneuvers that suppress conductivity in atrioventricular (AV) node such as Valsalva maneuvers, immersion of the face in cold water, etc. If non-medical measures fails, preferred medical management is with intravenous adenosine (preferred) or verapamil as the first-line medications.

Option A: Digoxin is no more recommended for management of PSVT.

Option B and C: Quinidine and flecainide are antiarrhythmic drugs, but not used for management of PSVT.

Option E: Beta blockers such as metoprolol, propranolol and atenolol are second-line drugs.

How well did you know this?
1
Not at all
2
3
4
5
Perfectly
31
Q

A 78-year-old man presents to your clinic with a 3-week history of palpitation. On examination, he has a blood pressure of 135/80 mmHg and heart rate of 115 bpm and irregular. ECG reveals atrial fibrillation (AF). An echocardiography is arranged that shows an ejection fraction (EF) of 38%. Which one of the following is the most appropriate treatment option for him?

A. Electrical cardioversion.
B. Metoprolol.
C. Digoxin.
D. Flecainide.
E. Verapamil.

A

B. Metoprolol.

The main goals of treatment of patients with AF + heart failure are control of symptoms and prevention of arterial thromboembolism. In patients with heart failure, hemodynamic consequences of AF results in decreased exercise capacity and the decompensation of heart failure.

Similar to the general population either rate control or rhythm control are mainstay of therapy. Previously, rhythm control was considered the superior method for treatment of AF in patients with heart failure; however, recent data has challenged this approach. Currently, rate control is the most appropriate initial management in patients with AF and symptoms related to rapid ventricular response. Rate control to prevent rapid AF often leads to an improvement in symptoms in patients with heart failure. Moreover, slowing of the ventricular rate often leads to a moderate or even marked improvement in left ventricular function. Based on current evidence, cardiac-selective beta blockers (metoprolol, atenolol, carvedilol, bisoprolol) are preferred options to use for rate control in patients with AF and heart failure because they are also recommended for treatment of heart failure itself. Digoxin can be used as adjunctive therapy if beta blockers fail to adequately control the rate.

If rate control cannot be achieved with beta blockers or combination of beta blockers and digoxin, amiodarone may be effective either alone or in combination with other rate-slowing medications.

NOTE - Initiation or increase of beta blocker is contraindicated in patients with decompensated heart failure. If such patients need rate, digoxin is the recommended agent; however, digoxin often fails to control the rate when used alone, especially in patients with elevated sympathetic tone (hypertension, tachycardia, etc.).

Beta blockers are contraindicated if any of the following is present:
* Heart rate < 60 bpm
* Symptomatic hypotension
* Greater than minimal evidence of fluid retention: increased jugular venous pressure, pulmonary crackles indicative of interstitial pulmonary edema, presence of S3, S4 or both, or a new or changed murmur
* Signs of peripheral hypoperfusion
* PR interval>0.24seconds
* Second- or third-degree AV block
* History of asthma or reactive airways
* Peripheral artery disease with resting limb ischemia

NOTE - The non-dihydropyridine calcium channel blockers verapamil and diltiazem are not usually recommended due to risk of an exacerbation of heart failure. If their use is considered, extreme caution is required.

This patient has both AF and heart failure; however, there is no finding in history or on examination to indicate that his heart failure is decompensated. On the other hand, he has no contraindication to beta blockers. A reduced ejection fraction of 38% alone is not a contraindication for beta blockers. In fact, beta blockers, by reducing heart rate and cardiac oxygen demand, will add to benefits of their use in this patient for rate control as the most appropriate management option.

Option A: Electrical cardioversion is used as an initial emergency treatment only if any of the 4 following conditions are present:
* Active ischemia (symptomatic or electrocardiographic evidence)
* Evidence of organ hypoperfusion
* Severe manifestations of heart failure including pulmonary edema
* The presence of a pre-excitation syndrome that may lead to an extremely rapid ventricular rate due to the presence of an accessory pathway

This patient has none of the above conditions to necessitate such treatment.

Option C: Digoxin is the initial management of patients with AF and decompensated heart failure. This patient’s heart failure is not decompensated. Even so, digoxin is not likely to slow the rate in the first hours of treatment.

Option D: Flecainide is an option for conversion to or maintenance of sinus rhythm and is used for rhythm control. Unless the patient has severe symptoms caused by the arrhythmia rather than the rapid rate, cardioversion should be performed on an elective basis after adequate anticoagulation has been achieved.

Option E: Verapamil is a non-dihydropyridine calcium channel blocker. These agents are appropriate options for rate control in the general population but should be avoided in heart failure due to significant risk of myocardial suppression and worsening of the heart failure associated with their use.

In short,
* AF - CCB (verapamil, diltiazem)
* AF + HF - BB +/- digoxin … fail then amiodarone
* AF + DHF - Digoxin
* Severe AF - Cardioversion then add flecainide

Main Goals in Treating AF + Heart Failure:
- Control symptoms
- Prevent arterial thromboembolism

Hemodynamic Impact of AF in Heart Failure:
- Decreased exercise capacity
- Decompensation of heart failure

Therapy Options:
- Rate Control: Preferred initial management for patients with AF and rapid ventricular response.
- Beta Blockers: Cardio-selective beta blockers (metoprolol, atenolol, carvedilol, bisoprolol) are preferred because they also treat heart failure.
- Digoxin: Used if beta blockers alone are not effective.

  • Rhythm Control: Previously favored but recent data suggest rate control is better for initial management.

If Rate Control Fails:
- Amiodarone: Effective alone or with other medications.

Notes on Beta Blockers:
- Contraindicated in decompensated heart failure.
- Do not use if:
- Heart rate < 60 bpm
- Symptomatic hypotension
- Significant fluid retention
- Signs of hypoperfusion
- PR interval > 0.24 seconds
- Second- or third-degree AV block
- History of asthma or reactive airways
- Peripheral artery disease with resting limb ischemia

Non-Dihydropyridine Calcium Channel Blockers:
- Verapamil and diltiazem are generally not recommended due to the risk of exacerbating heart failure.

Case-Specific Notes:
- Patient has AF and heart failure but no signs of decompensation.
- No contraindications to beta blockers.
- Beta blockers are beneficial even with a reduced ejection fraction (38%).

Management Options:
- Electrical Cardioversion: Only for emergencies if:
- Active ischemia
- Organ hypoperfusion
- Severe heart failure symptoms
- Pre-excitation syndrome

  • Digoxin: Used for decompensated heart failure but not likely effective immediately.
  • Flecainide: Used for rhythm control, not rate control.
  • Verapamil: Avoid in heart failure.

Summary for Treatment Choices:
1. AF: Calcium channel blockers (verapamil, diltiazem)
2. AF + Heart Failure: Beta blockers +/- digoxin, if needed, amiodarone
3. AF + Decompensated Heart Failure: Digoxin
4. Severe AF: Cardioversion, then add flecainide if necessary

Super long explanation - to refine later on*

How well did you know this?
1
Not at all
2
3
4
5
Perfectly
32
Q

A 65-year-old man presents to the emergency department with complaints of palpitation for the past 3 days which is worse today. He is on thyroxin 50 mcg for hypothyroidism. On examination, he has a blood pressure of 140/90 mmHg and an irregular pulse. An ECG is obtained and is as shown in the picture. Which one of the following would be the next step in management?

A. Stop thyroxin.
B. Start him on metoprolol.
C. Arrange for measuring plasma TSH level. D. Give him intravenous atropine.
E. Place a percutaneous pacemaker.

A

B. Start him on metoprolol.

The clinical findings of palpitation and irregular pulse in conjunction with ECG establish the diagnosis of atrial fibrillation (AF). AF may be idiopathic or due to a variety of conditions, with chronic hypertension being the most common cause. Hyperthyroidism is another etiology for AF. For every patient with AF, TSH should be measured (option C) and an echocardiography performed, but these steps are considered once the patient is started on appropriate treatment. For patients with symptomatic AF, treatment either with rate control (preferred) with metoprolol or calcium channel blockers or rhythm control should be started immediately.

Considering anticoagulation for prevention of thromboembolic events is another important step for AF with more than 48-hour duration or of unknown duration. Investigation for reversible causes of AF is considered once the patient is given appropriate initial management.

Option A: A TSH level below the normal lower limit is associated with an increased risk of AF, as is hyperthyroid state associated with excess doses of thyroxin. Even in the presence of increased levels of T4 in the plasma due to excess thyroxin, stopping the thyroxin will not lead to rapid improvement of symptoms.

Option D and E: Atropine and pacemakers are used for treatment of symptomatic bradycardia, which is not the case here.

How well did you know this?
1
Not at all
2
3
4
5
Perfectly
33
Q

A 69-year-old man just had a syncopal episode in the background history of aortic valve stenosis. He is now in the Emergency Department. Which one of the following is the most immediate management?

A. Electrocardiography.
B. Electrocardiography and cardiac monitoring.
C. Chest X-ray.
D. Emergency echocardiography.
E. Stress test.

A

B. Electrocardiography and cardiac monitoring.

Every patient over the age of 65 with an underlying cardiac disease is at risk of developing fatal arrhythmias such as complete heart block, ventricular tachyarrhythmias, etc. Although aortic stenosis is frequently associated with episodes of loss of consciousness and falls due to decreased perfusion on activity, the patient’s condition should not be merely attributed to this pathophysiology. An ECG should be obtained and the patient placed on cardiac monitoring for detection of potential arrhythmias that might have not been present when the ECG was taken.

Option A: ECG alone may miss the causative arrhythmia if it is transient.

Option C: Chest X-ray is not useful in detecting physiologic or structural heart anomalies.

Option D: Echocardiography may be considered later for additional pieces of information but not urgently. It has no role in detecting an arrhythmia. On the other hand, the diagnosis of aortic stenosis as a possible cause of syncope is already made.

Option E: Stress test is not helpful for detection of an arrhythmia. It can cause myocardial ischemia in this patient due to aortic stenosis.

How well did you know this?
1
Not at all
2
3
4
5
Perfectly
34
Q

A 62-year-old woman is seen in the ward 24 hours after a non-ST segment elevation myocardial infarction (NSTEMI). She complains of light- headedness. The radial pulse is difficult to feel, blood pressure is 90/60 mmHg, heart sounds are muffled, and there is evidence of biventricular cardiac failure. Which one of the following rhythm strips would be of most concern in this situation?

A

Correct Answer: A

The clinical picture of weak pulses, tachycardia and lightheadedness are suggestive of a tachyarrhythmia of ventricular origin, i.e. **ventricular tachycardia (VT) **or ventricular fibrillation (VF). The latter is always associated with loss of consciousness; therefore, cannot be considered in this woman.

The only rhythm strip consistent with the clinical picture is option A, which shows VT. VT is the most common arrhythmia occuring after myocardial infarction (MI). They are in particular common in the first 24 hours post-MI.

How well did you know this?
1
Not at all
2
3
4
5
Perfectly
35
Q

By definition, which one of the following best describes the condition termed as acute coronary syndrome?

A. Non-ST elevation myocardial infarction and ST elevation myocardial infarction immediately following cardiac surgery.
B. Unstable angina, non-ST elevation myocardial infarction and ST elevation myocardial infarction.
C. Stable angina, unstable angina and non-ST elevation myocardial infarction.
D. Stable angina, unstable angina and ST elevation myocardial infarction.
E. Unstable angina.

A

ACS consists of (option B.) unstable angina, non-ST elevation myocardial infarction and ST elevation myocardial infarction. They all present with similar clinical symptoms.

How well did you know this?
1
Not at all
2
3
4
5
Perfectly
36
Q

Which one of the following is not associated with prolongation of QT interval?
A. Hypothyroidism.
B. Hypercalcemia.
C. Haloperidol.
D. Sotalol.
E. Methadone.

A

B. Hypercalcemia.

Hypercalcemia affects the QT interval by shortening it, not prolonging it. Here’s a brief explanation for each condition:

  1. Hypothyroidism: Can lead to prolonged QT interval due to metabolic changes affecting the heart’s electrical activity.
  2. Hypercalcemia: Increases calcium levels in the blood, which accelerates the repolarization phase of the heart’s electrical cycle, leading to a shortened QT interval.
  3. Haloperidol: An antipsychotic that blocks potassium channels, delaying repolarization and prolonging the QT interval.
  4. Sotalol: A beta-blocker and antiarrhythmic that blocks potassium channels, leading to prolonged repolarization and QT interval.
  5. Methadone: An opioid that can block potassium channels, delaying repolarization and causing QT interval prolongation.

So, while conditions like hypothyroidism, and drugs like haloperidol, sotalol, and methadone prolong the QT interval, hypercalcemia does the opposite by shortening it.

The following is a list of conditions that can cause QT prolongation:
* Hereditary syndromes (rare)
* Electrolyte/metabolic abnormalities including hypocalcemia, hypokalemia or hypomagnesemia Intrinsic cardiac disease
* Medications:
Antiarrhythmics: amiodarone, disopyramide, dofetilide, ibutilide, procainamide, quinidine, sotalol Antihistamines: astemizole, terfenadine
Antibiotics: clarythromycin, erythromycin, pentamidine, sparfloxacin
Anti-malarials: chloroquine, halofantrine
Anti-psychotics: chlorpromazine, haloperidole, mesoridazine, pimozide, thioridazine Gastrointestinal drugs: cisapride, domperidone
Opiate agonists: methadone, levomethadyl
Other: Arsenic trioxide, bepridil, droperidol, probucol
* CNS disorders Systemic illnesses Myocardial Infarction
* Hypercalcemia is associated with shortening of QT interval and does not cause QT interval prolongation.

How well did you know this?
1
Not at all
2
3
4
5
Perfectly
37
Q

A 60-year-old man presents to the Emergency Department with complaint of vague abdominal pain. On examination, there is mild abdominal tenderness in the epigastric area with a vaguely palpated abdominal mass. On auscultation, a bruit is heard over the mass. Which one of the following is the next step in management?

A. Aortogram.
B. Emergency bedside ultrasonography.
C. Non-contrast abdominal CT scan.
D. Erect and supine abdominal X-rays.
E. Immediate transfer of the patient to the operating room.

A

B. Emergency bedside ultrasonography.

The findings are consistent with provisional diagnosis of abdominal aortic aneurysm (AAA). Physical examination is only moderately sensitive for diagnosis; therefore, a bedside ultrasonography for confirming the diagnosis of AAA is the next best step in management. Ultrasonography is the preferred diagnostic method for AAAs with high accuracy.

Spiral CT scan gives visualization of the aneurysm and surrounding structures. It is only indicated prior to elective surgical repair. It is never used to make a diagnosis of AAA. CT angiography with intravenous contrast can be used for assessment, but non-contrast CT and abdominal X-rays are of no use to diagnose AAA.

How well did you know this?
1
Not at all
2
3
4
5
Perfectly
38
Q

A 55-year-old man with history of smoking and hypertension has been diagnosed with a 4.2 cm abdominal aortic aneurysm (AAA). Which one of the following is the investigation of choice for monitoring and surveillance of his AAA?

A. MRI angiography.
B. CT angiography.
C. Ultrasonography.
D. Digital subtraction angiography.
E. Non-contrast abdominal CT scan.

A

C. Ultrasonography.

Ultrasonography is the most accurate and the best investigation for diagnosing, screening, and surveillance of aortic abdominal aneurysms Ultrasonography is a noninvasive and readily available test with approximately 100% sensitivity and specificity. CT angiography is performed if elective surgical correction is planned, once the diagnosis is made by ultrasonography.

How well did you know this?
1
Not at all
2
3
4
5
Perfectly
39
Q

A 55-year-old man presents with concerns about developing abdominal aortic aneurysm (AAA), as his father was diagnosed with the same disease at the age of 57 years. He has smoked 20 cigarettes a day for the past 15 years. On examination, he has a blood pressure of 130/75 mmHg. Which one of the following is the best advice to give?

A. Reassure as AAA has no genetic predisposition.
B. Arrange for regular screening with ultrasonography.
C. Order a CT angiogram.
D. Start him on aspirin.
E. Start him on anti-hypertensive medications.

A

B. Arrange for regular screening with ultrasonography.

Patients with a first- degree relative with abdominal aortic aneurysm (AAA) have a 20% increased risk of developing AAA compared to normal population; for this reason, he needs to be screened for AAA. Screening is performed with ultrasonography as the best investigation with very high index of sensitivity and specificity.

NOTE - Advancing age is the most common risk factor for AAA. Other risk factors include male gender, family history, smoking, hypercholesterolemia, diabetes mellitus, and chronic hypertension.

Option A: Reassurance is inappropriate because he has increased risk of AAA due to the family history.

Option C: CT angiogram is indicated once the surgical repair is planned.

Option D: Aspirin is used neither for prophylaxis, nor treatment of AAA.

Option E: Anti-hypertensive medications are of no benefit because the patient is normotensive.

40
Q

A 65-year-old man presents with history of two episodes of abdominal pain after meals. As a part of workup plan, an abdominal ultrasonography is performed, which showed gallstones in the gallbladder and a 3.8 cm abdominal aortic aneurysm (AAA). Which one of the following is the recommended management of this asymptomatic AAA?

A. Ultrasonography every 12 months.
B. CT scan every 12 months.
C. Ultrasonography every 24 months.
D. CT scan every 24 months.
E. CT scan or ultrasonography every 12 months.

A

C. Ultrasonography every 24 months.

Ultrasonography is the preferred method for screening and surveillance of asymptomatic AAA. The screening intervals depend on the size of the AAA and are according to the following table.

Given the size of the AAA (3.8 cm), it should be monitored every 24 months with ultrasonography.

Reference: RACGP - Aortic aneurysms Screening, surveillance and referral
41
Q

Abdominal ultrasonography of a 68-year-old man for renal colic reveals an abdominal aortic aneurysm 4.5 cm in size as a chance finding. Which one of the following is the most appropriate management of his aneurysm?

A. Refer him for elective surgical repair of the aneurysm.
B. Repeat sonography in 6 months.
C. Repeat sonography in 12 months.
D. Repeat sonography in 24 months.
E. Arrange for CT angiography.

A

C. Repeat sonography in 12 months.

Ultrasonography is the preferred method for screening and surveillance of asymptomatic AAA. The screening intervals depend on the size of the AAA and are according to the following table.

Given the size of the AAA (4.5 cm), it should be monitored every 12 months with ultrasonography.

RACGP - Aortic aneurysms Screening, surveillance and referral
42
Q

Which one of the following is not an indication for surgical repair of an abdominal aortic aneurysm (AAA)?

A. AAA more than 5.0 cm in a male patient.
B. AAA more than 5.0 cm in a female patient.
C. AAA growth more than 1.0 cm/year.
D. AAA with back pain.
E. AAA with distal thromboembolism.

A

A. AAA more than 5.0 cm in a male patient.

The following are indications for surgical repair:
* Asymptomatic AAA greater than over 5.5cm in men
* Asymptomatic AAA over 5.0cm in women
* Thoracic aortic aneurysm over 6.0cm
* Growth of more than 1.0cm/year
* Symptomatic AAA (abdominal/back pain, distal thromboembolism)

An AAA of 5.0 cm in a male is not an indication for surgical repair.

43
Q

A 72-year-old man comes to the emergency department complaining of epigastric pain that radiates to his lower back. While waiting for further evaluation in the emergency department, he suddenly collapses. Physical examination shows a blood pressure of 80/50 mmHg and pulse of 110 bpm. He is pale, cold, and sweaty. Which one of the following is the next best step in management?

A. Take the patient to the operating theatre immediately.
B. Arrange for CT angiography.
C. Non-contrast abdominal CT scan.
D. ECG.
E. Bedside ultrasonography.

A

E. Bedside ultrasonography.

The location of the pain and its radiation to the lower back, followed by signs of shock is consistent with a ruptured abdominal aortic aneurysm as the most likely provisional diagnosis. Before emergent transfer of the patient for surgery (option A.) , vascular surgeon should be contacted, and the diagnosis should be confirmed with bedside ultrasonography as the most readily available with acceptable sensitivity and specificity. In the meanwhile, resuscitative measures such as intravenous access, fluid administration, and supplemental oxygen should be taken in the emergency department. An ECG (option D.) must be obtained pre-operatively but not the priority as the first step in management.

Other diagnostic measures such as CT angiography (option B.) or non-contrast CT (option C.) scan are unnecessary and does not change the management plan.

44
Q

You are called to see a 58-year-old patient, who has presented with complaints of weakness and shortness of breath. A blood test reveals hemoglobin of 85 g/L (120-160). As a part of the workup for finding the cause of his anemia, an abdominal CT scan is performed showing a 10.5 cm infra-renal abdominal aortic aneurysm as an incidental finding. You tell the patient that he needs surgery for treatment of his aneurysm. He wants to know how dangerous it could be if he does not have the surgery. Which one of the following is the mortality rate of a ruptured abdominal aortic aneurysm?

A. 10%.
B. 25%.
C. 50%.
D. 80%.
E. 100%.

A

D. 80%.

Rupture of an AAA into the peritoneal cavity is often usually rapidly fatal, whereas, retroperitoneal rupture often stabilizes transiently providing a window of opportunity for life-saving surgical intervention and repair. Collectively, a ruptured AAA is catastrophic and carries a mortality risk of 80-90%.

45
Q

A 72-year-old man is brought to emergency department with ambulance after he sustained a sudden severe chest pain radiating to his back. On auscultation there is a diastolic murmur best heard over the left sternal border in the second intercostal space. ECG shows ST elevation in leads II, III and aVF. A CXR is remarkable for a widened mediastinum. Which one of the following is the most appropriate management?

A. Give alteplase.
B. Give aspirin, intravenous morphine, and beta blockers.
C. Give morphine and beta blockers and arrange for emergency trans-esophageal sonography.
D. Refer the patient to cardiology clinic.
E. Arrange for emergency angiography.

A

C. Give morphine and beta blockers and arrange for emergency trans-esophageal sonography.

The clinical findings of abrupt chest pain radiating to back and a widened mediastinum on CXR are consistent with diagnosis aortic dissection. The diastolic murmur signifies involvement of the proximal aorta resulting in aortic regurgitation and decreased blood flow to the coronary arteries and a consequent myocardial infarction (MI).

Option A: Thrombolytics (e.g. alteplase) are absolutely contraindicated when aortic dissection is suspected.

Aspirin (Option B.) , clopidogrel and other anticoagulation medications are contraindicated as well due to increased risk of deterioration of the dissection, which is temporarily stable.

The cornerstone of initial therapy in this situation is morphine for pain control (and its partial effect on lowering blood pressure) and beta blockers to maintain the systolic blood pressure below 120 mmHg. Trans-esophageal sonography should be emergently performed to confirm the diagnosis. Angiography using CT angiogram (CTA) (Option E.) is the preferred method if trans- esophageal sonography is not available.

Option C: Referring the patient to a cardiology clinic while he is in the hopsital and in urgent need for intervention is not an appropriate option.

46
Q

Which one of following is the most common cause of acute coronary syndrome?

A. Cocaine toxicity and consequent coronary artery vasospasm.
B. Acute thrombosis.
C. Chest trauma.
D. Arterial inflammation.
E. Anxiety.

A

B. Acute thrombosis.

Atherosclerotic changes of coronary arteries remain the most common cause of ischemic heart disease. The most common cause of acute coronary syndrome is acute rupture of an atheroma followed by platelet aggregation and thrombus formation. Although other options can lead to acute coronary syndrome, they are far less common.

47
Q

Which one of the following is the most common cause of acute limb ischemia?

A. Congestive heart failure.
B. Thrombosis.
C. Vasculitis.
D. Embolism.
E. Smoking.

A

B. Thrombosis.

Thrombotic occlusion is the most common cause of acute lower limb ischemia accounting for more than 80% of cases.

Option D: Occlusion from embolus is not as common, partly due to decline in rheumatic heart disease and prompt management of patients with atrial fibrillation with prophylactic anticoagulation. Atrial fibrillation accounts for two-thirds of acute limb ischemias due to embolism.

Option A, C and E: Congestive heart failure, vasculitis and smoking are less common risk factors of acute limb ischemia.

48
Q

Which one of the following signs mandates emergency surgical intervention in acute limb ischemia?

A. Pallor.
B. Coldness.
C. Paralysis.
D. Pain at rest.
E. Pulselessness.

A

C. Paralysis.

Paralysis is present when a patient with acute limb ischemia is unable to wriggle the toes or fingers. It is the most reliable sign indicating acute limb ischemia requiring emergency surgical intervention.
Paresthesia of the affected limb is another indicator of acute limb ischemia
and can be used in the absence of paralysis.

Option A: Pallor is among the signs suggesting acute limb ischemia; however, the color of the limb may be affected by the ambient light.

Option B: Coldness of the limb is the least reliable sign as it can be due to decreased body temperature of the patient to cold weather.

Option D: Pain (either with exertion or at rest) and pallor can be caused by chronic ischemia as well, and are not reliable indicators of the need for emergency surgical treatment.

Option E: Pulselessness can be a sign of acute limb ischemia, but paralysis remains the most critical indicator.

49
Q

A 65-year-old man presents to the emergency department with sudden onset right-sided leg pain and paresthesia. On physical examination, the distal pulses of the right leg are absent and the limb is cold. A CT angiography is arranged showing thrombotic occlusion of the right femoral artery. Which one of the following is the next best step in management?

A. Intravenous infusion of heparin and emergency embolectomy.
B. Intravenous infusion of heparin for 24 hours and review.
C. Complete bed rest with compression bandage and leg elevation. D. Start the patient on warfarin.
E. Start the patient on aspirin and clopidogrel.

A

A. Intravenous infusion of heparin and emergency embolectomy.

The clinical and imaging findings are quite consistent with acute limb ischemia. Paralysis, paresthesia and compartment syndrome are ominous signs that demand emergency surgical intervention after heparin has been started. The golden time for surgical intervention is 4 hours. Signs of ischemia are are reversible if prompt action is taken within this period. Prolonged acute ischemia(>6 hours) leaves irreversible and permanent deficits.

Heparin cover should be maintained and warfarin started. Heparin then can be safely withheld once the INR is 2-3. Warfarin alone is not recommended as it is pro-coagulation at the beginning.

50
Q

A 70-year-old man presents with acute pain and paralysis of the right leg diagnosed to have been caused by acute leg ischemia. Heparin is started immediately. After emergency imaging, he is transferred to the operating room for embolectomy. Surgical intervention successfully restored blood supply to the affected limb after 3 hours. This patient is at risk of developing reperfusion injury as a result of prolonged ischemia. Which one of the following is not a characteristic feature of reperfusion injury?

A. Hyperkalemia.
B. Hypokalemia.
C. Metabolic acidosis.
D. Myoglobinuria.
E. Elevate creatine kinase.

A

B. Hypokalemia.

Reperfusion injury is a complication of blood restoration to a limb, which has been ischemic for a while. Characteristic features of reperfusion injury are all related to ischemia and its impacts on tissue (especially muscle cells).

Features of reperfusion syndrome include:
* Metabolic acidosis (lactic acidosis)
* Elevated creatine kinase
* Hyperkalemia
* Myoglobinemia and myoglobinuria

These findings are caused by hypoxemia resulting in metabolic acidosis, and muscle cell breakdown and release of its cell into the blood. Hypokalemia is not a characteristic feature of reperfusion injury.

The extent of the reperfusion injury depends on the following:
* Duration and the site of arterial blockage
* The extent of collateral flow to the affected area
* The previous health of the affected limb

Approximately one third of all deaths from arterial occlusions are due to metabolic complications after revascularisation.

Reperfusion Injury after Blood Flow Restoration

When blood returns to a limb after a period of reduced flow (ischemia), it can lead to reperfusion injury. Key features include:

  • Metabolic Acidosis (Lactic Acidosis): Lack of oxygen during ischemia causes cells to produce lactic acid, leading to acidosis when blood flow returns.
  • Elevated Creatine Kinase: Enzyme released from damaged muscle cells.
  • Hyperkalemia: High potassium levels in the blood due to cell breakdown.
  • Myoglobinemia and Myoglobinuria: Release of muscle protein into the bloodstream and urine.

These complications arise because ischemia starves tissues of oxygen and nutrients, leading to cell damage. Reperfusion worsens this by reintroducing oxygen suddenly, causing stress to already compromised cells.

Factors influencing the severity of reperfusion injury include:
- Duration and Location of arterial blockage.
- Collateral Blood Flow to the affected area.
- Previous Health of the limb.

About one-third of deaths from arterial blockages result from metabolic issues after restoring blood flow.

51
Q

A 55-year-old man presents to the emergency department after sudden onset of pain and paresthesia in his left leg. On examination, the distal pulses of the left lower limb are absent and the limb is cold and painful. A CT angiogram is performed and establishes the presence of femoral artery embolism. The patient was started on intravenous heparin and embolectomy was carried out. Which one of the following is the next best step in management?

A. Aspirin and warfarin for 6 months.
B. Lifelong aspirin.
C. Warfarin for 3-6 months.
D. Low molecular weight heparin for six months and lifelong warfarin afterwards.
E. Heparin for one week.

A

C. Warfarin for 3-6 months.

Every patient with acute limb ischemia must be started on anticoagulation therapy with warfarin after embolectomy is performed. Heparin cover should not be withheld until the INR is in the therapeutic range of 2-3.

Long-term warfarin should be considered following embolism from a cardiac source; otherwise 3-6 months warfarin may suffice.

Aspirin is indicated in patients with atherosclerotic changes of the peripheral arteries and chronic limb ischemia. It has no role in management of acute limb ischemia.

Combination of aspirin and warfarin is not shown to benefit the patient more than warfarin alone in terms of preventing further thromboembolic events. Furthermore using these two drugs in combination increases the risk of bleeding.

Heparin is only used until the INR is in the therapeutic range and withheld afterwards. This goal is usually achieved in 2-3 days.

52
Q

A 55-year-old man is on heparin after he developed an acute limb ischemia. Which one of the following tests is used for monitoring the response to anticoagulation therapy with heparin?

A. Fibrinogen level.
B. Prothrombin time (PT).
C. INR.
D. Activated partial thromboplastin time (aPTT).
E. Bleeding time (BT).

A

D. Activated partial thromboplastin time (aPTT).

Activated partial thromboplastin time (aPTT) is used to assess the integrity of the following coagulation pathways:
* Intrinsic pathway that includes prekallikrein, high molecular weight kininogen and factors VIII, IX, XI, and XII.
* Common pathway including factors II, V, X, and fibrinogen.

aPTT is also used for monitoring the effects of maintenance therapy with heparin. Usually the goal of heparin maintenance therapy is to keep aPTT level 1.5- 2.5 times above patient’s aPTT baseline. When starting heparin therapy, aPTT is measured 6-hourly until it is within therapeutic range of 65-100 seconds, and 24 hourly afterwards.

INR is used for monitoring warfarin therapy. It assesses extrinsic coagulation pathway, as does PT. Normal range of PT in an individual not on blood thinners is 11-13.5 seconds. PT and INR are prolonged if the patient is on warfarin, or deficient in factors II (prothrombin), III, IX, X, or fibrinogen, or has vitamin K deficiency.

BT is used as a platelet function test with a normal range of 1-9 minutes. It is prolonged if the patient is on antiplatelet medications (aspirin, clopidogrel), has thrombocytopenia, or in those with platelet aggregation disorders e.g. Von-Willebrand disease. BT is not used routinely anymore.

53
Q

A 65-year-old man comes to your practice with complaint of pain in buttock and leg. He has the pain for the past three months, and describes it as cramping felt in the left buttock. The pain is only present when he starts walking and comes on after five minutes or so every time. It radiates to the left thigh and goes away few minutes after he stops walking. Which one of the following is the most likely diagnosis?

A. Neurogenic claudication resulting from spinal canal stenosis.
B. Stenosis of his left common iliac artery.
C. Stenosis of the left superficial femoral artery.
D. Osteoarthritis of the right hip joint.
E. L4/L5 disc prolapse.

A

B. Stenosis of his left common iliac artery.

The nature of the pain (cramping) and its pattern (brought on by walking, relieved by rest) is characteristic of chronic limb ischemia. Exertional buttock pain that is ischemic in nature (buttock claudication) is due to obstruction of either common iliac or external iliac artery.

Option A: Spinal canal stenosis usually affects the mid or lower lumbar spine and can cause nerve root impingement presentation, which is pain brought on by standing and relieved by recumbency. Pain distribution is related to the nerve root distribution rather than muscle groups supplied by an involved artery.

Option C: Stenosis of the superficial femoral artery gives rise to leg rather than buttock claudication.

Option D: Osteoarthritis of the hip is associated with pain on activity and relieved by rest. The pain is felt in the hip and can radiate to the knee. Buttock pain is not common.

Option E: L4/L5 disc prolapse can result in unilateral nerve root entrapment leading to neurological findings, not necessarily related to walking. The pain of L4/L5 disc prolapse is worse on lifting heavy objects, coughing, sneezing or straining at stool. Numbness and paresthesia are features that are commonly present in nerve root impingement, but not in chronic limb ischemia.

54
Q

A 76-year-old man comes to your office for evaluation. He mentions that he has difficulty walking because of the left leg pain. The pain is brought on after walking two blocks and gets better when he stops to rest. On examination, the leg skin is shiny and dark. The legs hair is lost and the muscles are atrophied. Distal pulses are difficult to palpate. Which one of the following is the most likely diagnosis?

A. Acute limb ischemia.
B. Deep vein thrombosis.
C. Superficial vein thrombosis.
D. Chronic obstructive arterial disease.
E. Spinal canal stenosis.

A

D. Chronic obstructive arterial disease.

Leg pain brought up by walking and relieved by rest and weak or absent distal pulses are characteristic of chronic limb ischemias as a result of chronic obstructive arterial disease. Atrophied muscles and shiny hairless skin supports the diagnosis.

The clinical findings in chronic limb ischemia include:
* Weak or absent distal pulses – the hallmark finding
* Shiny and hyperpigmented skin
* Hair loss and leg ulcers
* Thickened nails
* Muscular atrophy
* Vascular bruits

Option A: Acute limb ischemia presents with sudden onset pain, pallor, paralysis, paresthesia, pulselessness and poikilothermia. This patient has features of chronic limb ischemia.

Option B: Leg pain due to deep vein thrombosis can be brought on by walking and relieved by rest (similar to chronic limb ischemia), but other features such as sparse leg hair, pigmentation, muscle atrophy, etc are not feautres of DVT. DVT presents with leg pain and tenderness, swelling and warmth.

Option C: Superficial thrombophlebitis presents with pain, erythema, induration and tenderness along the course of a superficial vein.

Option E: Leg pain caused by neurogenic claudication due to spinal canal stenosis tends to get worse with erect posture and relieved by recumbency. Absence of neurological deficits makes this diagnosis less likely.

55
Q

A 60-year-old man presents with leg pain for the past 6 months. The pain becomes worse with walking and is relieved when he rests. There is no pain at rest. He has smoked 20 cigarettes per day for the past 30 years. On examination, he is obese with a BMI of 31 and has a blood pressure of 160/110 mmHg on two readings 20 minutes apart. Distal pulses of the left lower limb, including dorsal pedis, are barely perceptible. The skin of the legs is shiny and hairless. Mild muscular atrophy of the leg is noted. Ankle-brachial index (ABI) is 0.7. Which one of the following would be the most appropriate management and advice?

A. Smoking cessation, exercise and follow-up in three months.
B. Smoking cessation, statins and ACE inhibitors.
C. Duplex Doppler venous ultrasonography.
D. Aspirin, metoprolol and statins.
E. Referral for vascular surgery.

A

B. Smoking cessation, statins and ACE inhibitors.

With intermittent calf claudication, presence of the risk factors and an ABI of 0.7, the diagnosis of chronic peripheral arterial disease is almost established. Management of peripheral arterial disease includes the following:

Smoking cessation - smoking is the most important predisposing factor for peripheral arterial disease (PAD). Smoking cesation alone is associated with an improvement in the distance of pain-free walk, doubled 5-year survival and better post-op outcomes.

Exercise – exercise on an as tolerated basis, improves the pain-free walking distance and time and should be advised for all patients.

ACE inhibitors – evidence suggests that ACE inhibitors may improve walking ability in patients with intermittent claudication. The ACE inhibitor with greatest evidence of benefit is ramipril. It is unknown if the improvement in walking distance associated with ramipril is due to a class effect of ACE inhibitors or whether it is specific to this medicine. The ABI does not seem to improved though.

Statins – statins improve revascularisation, pain-free walking distance and survival. Of all lipid-lowering agents, only statins have been proved to lower the mortality in patients with vascular diseases due to atherosclerotic changes. It should be started for patients with coronary artery disease, PAD, aortic disease (e.g. abdominal aortic aneurysm), carotid artery disease and diabetes mellitus.

Clopidogrel and aspirin – they are often prescribed to reduce the overall risk of myocardial infarction and stroke, but are not associated with improvement of PAD symptoms.

Beta blockers (e.g. metoprolol) are not indicated in the absence of cardiac disease. They have no effect on PAD.

More detailed management of PAD is highly dependent on the severity of the disease. ABI is the mostly used predictor for this purpose. ABI is interpreted as follow: (see attachement)

Among the given option, smoking cessation, statins, and ACE inhibitors are the best possible management and the advice for this patient.

56
Q

A 56-year-old man comes to your practice with an eight-month history of left leg pain, which is brought on by walking, and used to relieved with rest. Recently, he gets the pain even at rest, particularly at night. He has to dangle his leg for relative pain relief. He has smoked for the past 30 years and been on hydrochlorothiazide for the past 10 years. On examination, he is an overweight man with a BMI of 29, blood pressure of 145/100mmHg and pulse of 98bpm. His left leg is hairless with shiny and hyper-pigmented skin. The left femoral pulse is barely perceptible and the right dorsal pedis pulse cannot be felt. He has an ankle-brachial index (ABI) of 0.4 on the left side. Which one of the following is the most appropriate management?

A. Advise him to stop smoking, do more exercise and review in three months.
B. Advise him to stop smoking, do more exercise and start him on statin.
C. Arrange Duplex Doppler ultrasonography and review.
D. Urgent referral for vascular surgery.
E. Arrange for arteriography and review.

A

D. Urgent referral for vascular surgery.

The clinical picture is consistent with chronic peripheral artery disease (PAD) and limb ischemia suggested by calf claudication and the presence of risk factors (hypertension, smoking and obesity). The rest pain and ABI of 0.4 suggests a critical ischemia, for which surgical intervention is required to be planned; otherwise he will end up gangrene and amputation. This patient should be urgently referred to a vascular surgeon for assessment and treatment.

Option A and B: Smoking cessation is the most appropriate advice that should be given to patients with PAD. It increases the exercise tolerance and walking distant and slows progression to more advanced disease. Statins should be started for patients with PAD as well as those with coronary artery disease, diabetes mellitus, aortic diseases due to atherosclerosis, and carotid artery disease. Pentoxifyline (not an option) might have a role, but trials have shown moderate benefit with this drug.

Option C: Duplex Doppler ultrasonography has already been used for determination of the ABI. It is unable to locate the anatomy of the lesion and a road map for surgery. Duplex Doppler ultrasonography is used to assess the blood flow to the leg and calculation of ABI, and the presence of a clot in the venous system when DVT is suspected. It cannot accurately spot the site of occlusion in an artery.

Option E: Once surgical intervention is considered by a surgeon, the presence and site of occlusion should be indentified and documented prior to the surgery. This is achieved by either less invasive CT angiography or MRI angiography, or through more invasive procedure of conventional arteriography.

ABI <= 0.4 —- urgent referral!

57
Q

A 28-year-old man develops rapid palpitations and mild light-headedness following a 20km run. He denies any cardiac disease and mentions that a he had a similar episode 3 months ago for which he underwent cardiovascular examination by another doctor but everything was normal. On examination, he has regular but rapid pulse of 180 bpm. Which one of the following rhythm strips would be expected for him?

A

D. Paroxysmal supraventricular tachycardia (PSVT).

An arrhythmia induced by exercise in a young, otherwise healthy person, presenting with a rapid regular pulse is very likley to be a paroxysmal supraventricular tachycardia (PSVT).

The presence of ventricular tachycardia (option A) in the absence of an underlying cardiac disease is unlikely, as are complete heart block (option E) and atrial flutter (option C).

Atrial fibrillation (option B) presents with irregular pulse.

58
Q

You are assessing the risk of thromboembolism in a 75-year-old woman, who is booked for elective surgical replacement of her left knee joint. Which one of the following, if present in history, is the most significant risk factor for thromboembolism during her peri-operative stay?

A. History of colon resection due to toxic megacolon.
B. Irritable bowel syndrome.
C. Stage II kidney disease.
D. Transient ischemic attack with full recovery three years ago.
E. Heart failure.

A

E. Heart failure.

Peri-hospital risk assessment of a venous thromboembolic events (VTE) such as deep vein thrombosis or pulmonary embolism is stratified according to the following table:

Among the options, congestive heart failure and the surgery are of the highest risk for venous thromboembolism. None of the other options are risk factors for VTE.

59
Q

Five days after surgical resection of a colonic tumor, a 73-year-old man develops pain, swelling and tenderness in his right calf. Physical examination reveals swelling of the ankle and the calf and tenderness over the gastrocnemius muscle. Which one of the following is the most appropraite diagnostic tool to establish the diagnosis?

A. CT angiography of the leg.
B. MRI of the leg.
C. Conventional venography of the leg.
D. Serum D-dimer.
E. Duplex Doppler ultrasonography.

A

**E. Duplex Doppler ultrasonography.

With the clinical picture suggestive of deep vein thrombosis, Duplex Doppler ultrasonography should be used to confirm the diagnosis. Although pain, swelling, and tenderness are suggestive of DVT, but treatment should never be started solely on clinical basis.

Option A: CT angiography is used for visualization of clots in arteries not veins.

Option B: MRI is being used increasingly for assessment of DVT, and probably is the preferred investigation for suspected caval and iliac venous thrombosis, particularly in pregnancy, but is expensive, time-consuming and not always available.

Option C: Conventional venography is the criterion standard for diagnosis of DVT, but is rarely used now because less invasive and convenient diagnostic modalities are widely available.

Option D: D-dimer assay, although sensitive, lacks specificity. Increased levels of D-dimer can be seen in several conditions, with post-surgery state as one. It is basically used to rule out DVT or pulmonary embolism in low risk patients (Wells score < 2). A positive test demands more specified evaluation, but a negative one excludes the possibility of DVT or PE.

AMC handbook multiple choice questions – page 173

60
Q

A 73-year-old man presents with swelling of his left leg for the past three hours. Evaluation establishes the diagnosis of a deep vein thrombosis (DVT) in the left lower leg. A Doppler ultrasound reveals deep vein thrombosis in the calf as well as popliteal and femoral veins. His past medical history is negative for previous DVT. Which one of the following is the next best in management of this patient?

A. Angiography.
B. Angioplasty.
C. Heparin.
D. Warfarin.
E. Aspirin.

A

C. Heparin.

All patients with DVT are initially managed by injectable heparin (either intravenous unfractionated or subcutaneous low molecular weight heparin). Warfarin can be started within 24-48 hours with close monitoring of INR. Once INR is in the therapeutic range of 2-3, heparin can be withheld. Warfarin should be continued for 6 months at least. Patients should see their GP on a regular basis for close monitoring of the INR.

Option A: DVT is the presence of clot in the deep veins. Arteriography evaluates arterial structure and thromboses within arteries.

Option B: Angioplasty is performed for revascularizaion in arterial problems with no role in DVT.

Option D: Warfarin is indicated in this patient after initial anticoagulation with heparin.

Option E: Antiplatelet therapy with aspirin or clopidogrel does not provide adequate anticoagulation for prevention of DVT.

61
Q

An 82-year-old man presents to your office complaining of an ulcer over the medial side of his leg just above the internal malleolus. He mentions that the wound has been there for the past 6 months, and worsened recently. The examination confirms the diagnosis a venous ulcer. The ipsilateral ankle-brachial index is 0.7. Which one of the following is correct regarding the management of this patient?

A. Compression stockings are unsafe for this patient.
B. Prescribe compression stockings and refer him to a vascular surgeon.
C. Give antibiotics and review in two weeks.
D. Check fasting glucose.
E. Check HbA1C.

A

A. Compression stockings are unsafe for this patient.

Venous leg ulcers are the most common chronic wounds seen in general practice. They account for 70-90% of chronic wounds. They tend to develop over the medial distal leg (just above the internal malleolus), and are usually painful.

The most important risk factor for developmet of a venous ulcer is previous deep vein thrombosis (DVT) with consequent damage to venous structure and insufficiency.

Most venous ulcers respond adequately to a conservative management called Bisggard regimen. This regimen has four components:
1. Patient education
2. Elevation of the foot
3. Elastic compression
4. Evaluation

Elastic compression is considered for very patient unless:
* ABI is below 0.8
* The patient is diabetic

In the above situations, compression stocking causes more compromised arterial flow and makes the limb ischemia worse; therefore, specialist advice and close monitoring is strongly recommended.

Antibiotics are not indicated in uncomplicated venous ulcers. Diabetes is one of the most important predisposing factors in development of arterial ulcers. Although fasting glucose level may be considered for screening of diabetes, it is not a step in management here. HbA1C is used to monitor efficacy of or adherence to anti-hyperglycemic regimens, which is not the case here.

62
Q

A 55-year-old man presents to the Emergency Department with episodes of palpitation and light-headedness. An ECG is obtained and is inconclusive. He is placed on telemetry monitoring. Evaluation of the records shows nonsustained paroxysms of ventricular tachycardia. There are many of these paroxysms during the night. Which one of the following is the next best step in management?

A. Amiodarone.
B. Bisoprolol.
C. Verapamil.
D. Synchronized DC cardioversion.
E. Digoxin.

A

A. Amiodarone.

For this patient with symptomatic non-sustained VTs, Amiodarone is the drug of choice. Amiodarone is considered first-line treatment option for those with sustained VT who are hemodynamically stable, or those with symptomatic non-sustained VT. Sotalol and lignocaine are second- and third-line medications, respectively.

Option D: Synchronized cardioversion is the next best in management of patients with sustained VT and consequent hemodynamic instability i.e. chest pain, dyspnea, hypotension, and perfusion-related confusion. Pulseless VT should be treated similar to ventricular fibrillation (VF) with defibrillation.

In outpatient setting, beta blockers (atenolol, bisoprolol, or metoprolol) (option b) , orally, are used for nonsustained episodes of VT associated with symptoms as preferred options. Amiodarone, sotalol or flecainide, orally, are second-line options. Calcium channel blockers (verapamil) (option c) or digoxin (option e) are not used for management of ventricular tachycardia.

63
Q

A-79-year-old woman undergoes total hip replacement surgery. After 4 days, she develops a swollen right leg. The circumference of the right calf is 4 cm greater than that of the left leg. The right leg feels warm to touch and is slightly tender. Which one of the following is most appropriate diagnostic tool to use for a diagnosis?

A. Duplex Doppler ultrasound.
B. X-ray.
C. Contrast venography.
D. CT angiography.
E. D-dimer assay.

A

A. Duplex Doppler ultrasound.

This patient has suspected diagnosis of deep vein thrombosis (DVT) and requires urgent Doppler ultrasound to establish the diagnosis.

Doppler ultrasound is more than 90% sensitive and more than 95% specific for thromboses within the femoral and popliteal veins, but less accurate for iliac or calf vein thromboses.

D-dimer is used for assessment of patients with Well score of < 2.

A patient with symptoms and signs consistent with DVT should be assessed and the clinical (pretest) probability of acute DVT established by using a validated scoring system. If the clinical probability of DVT is low (< 2), a D-dimer assay should be performed; otherwise, the next step would be Doppler Duplex sonography.

See Modified Wells’ Score below.

This patient has been bedridden for more than 3 days (1 point), has localized tenderness (1 point) and the circumference of the affected leg is more than 3 cm greater than the unaffected leg (1 point). With a Wells’ score or 3, she has high probability diagnosis of DVT and needs Doppler ultrasonography of the leg as the next step.

If the Wells’ score was < 2 the next best step would have been D-dimer assay (option E). A negative D-dimer test excludes DVT (and pulmonary embolism), but positive values is only suggestive of DVT and further investigations would be needed.

Previously, contrast venography (option C) was the definitive test for the diagnosis of DVT but has been largely replaced by ultrasonography, which is noninvasive, more readily available; however, it still remains the gold-standard for diagnosis of DVT (not in practice). While a Doppler ultrasound can establish the diagnosis, there is no need for CT angiography (option D) unless accurated anatomical site of the thrombus is required for surgical intervention (very rarely indicated).

64
Q

A 55-year-old man presents to a regional hospital complaining of chest pain. ECG shows ST segment elevation in leads I, V1, V5 and V6. The nearest tertiary hospital is 300 km away. He has been given aspirin, oxygen, and morphine. Which one of the following is the most appropriate step in management of this patient?

A. Transfer the patient to a tertiary hospital for PCI.
B. Give him intravesnous tissue plasminogen activator.
C. Give him beta blockers.
D. Give him clopidogrel.
E. Give him glyceryl trinitrate.

A

B. Give him intravesnous tissue plasminogen activator.

With chest pain and ST elevation in two contiguous leads, the diagnosis of ST elevation myocardial infarction (STEMI) is established and emergency reperfusion therapy is indicated as the first-line management.

Reperfusion is best achieved with PCI if it can be performed within 90 minutes of presentation. In this case, however, the time needed for travelling 300km to a tertiary hospital for PCI is far beyond the 90-minute limit; therefore, reperfusion with fibrinolytics such as tissue plasminogen activator (tPA) is the best option.

Beta blockers, nitrates are part of the management plan but not the most appropriate step in this situation. Clopidogrel is best avoided if the patient is undergoing fibrinolytic therapy.

Topic Review on Page 72 of PDF

65
Q

A 40-year-old man presents to the emergency department after sudden onset of the right calf pain and paralysis. The right dorsal pedis pulse is not perceptible. The limb feels cold and is pale. The patient is given analgesics. After starting the patient on heparin, which one of the following would be the most appropriate step in management?

A. Abdominal CT.
B. CT angiogram.
C. Doppler Duplex ultrasound of the calf veins.
D. Pelvic ultrasound.
E. Echocardiography.

Artery or Vein?

A

B. CT angiogram.

This patient has developed critical acute limb ischemia requiring urgent vascular surgery for revascularization and restoration of blood supply. Clinical signs of acute arterial occlusion include (6 P’s):
* Pain
* Paralysis
* Pulselessness
* Pallor
* Paresthesia
* Poikilothermia.

With any of the above signs or symptoms the patient is considered to have acute limb ischemia.

To confirm the diagnosis of acute arterial occlusion and the extent of obstruction, the patient should have an urgent CT angiogram as the road map to the surgery. Magentic resonance angiography with contrast is an alternative to CT angiography with about the same sensitivity and specificitiy.

Option A: Abdominal CT scan is not required as this patient did not present primarily with abdominal problems.

Option C and D: Pelvic ultrasound or Doppler Duplex ultrasound of the calf veins adds nothing to the management strategy because firstly the diagnosis is already made, and secondly these modalities do not provide adequate information regarding the anatomical site of the occlusion. Furthermore, Doppler Duplex ultrasound is highly operator-dependent.

Option E: Echocardiography is the investigation to consider once acute phase of arterial occlusion has been managed. Thrombi from the heart is a main source of acute limb ischemia.

Artery

66
Q

A 45-year-old man comes to your practice for a routine health check. He has no remarkable past medical history. On examination, you feel his pulse is irregular with an average rate of 84 bpm. The rest of the exam is unremarkable. He denies any history of lightheadedness, shortness of breath, palpitation or chest pain. An ECG is ordered which is shown in the accompanying photograph. Which one of the following is the most appropriate next step in management of this patient?

A. Start him on metoprolol.
B. Start him on warfarin.
C. Start him on digoxin.
D. Start him on aspirin.
E. Do an echocardiography.

A

D. Start him on aspirin.

The ECG shows atrial fibrillation (AF) without tachycardia, and the given scenario is that of an asymptomatic AF. The priority in management of patients with asymptomatic AF centers around prevention of thromboembolic events, until elective cardioversion is performed either medically or by application of synchronized cardioversion. Patients with AF in the setting of valvular heart disease should be started on warfarin. Other patients with AF should be stratified and managed based on CHA2DS2 - VASc index.

C: Congestive heart failure / Left ventricular dysfunction (1)
H: Hypertension (1)
A2: Age ≥75 years (2) 
D: Diabetes mellitus (1)
S2: Previous history of stroke or TIA (2)
V: Vascular disease i.e., peripheral arterial disease, previous MI, aortic plaques (1)
A: Age 65-74 years (1)
Sc: Sex category (i.e. female sex) (1)

NOTE - maximum score is 9 since age may contribute 0, 1, or 2 points 9

This patient has a CHA2DS2 - VASc score of 0. For him both no anticoagulation phrophylaxis or anticoagulation therapy with low-dose aspirin are acceptable options. There is no option suggesting ‘no anticoagulation therapy’; therefore, the only acceptable option would be anticoagulation therapy with aspirin.

For symptomatic patients or older patients with no or mild symptoms, who have rapid response AF evident by tachycardia, rate control should be started with beta blockers (first-line) such as metoprolol (option A) or calcium channel blockers (second-line). Digoxin (option C) is no longer recommended for rate control.

Echocardiography (option E) and TSH are two most important steps considered for every patient with new onset or new diagnosed AF, but not as the next best step in management.

NOTE - *Rhythm control* (vs. rate control) may be considered in the following patients groups

  • Patients with unacceptable arrhythmia-related symptoms
  • Patients presenting for the first time with non-valvular disease
  • AF secondary to an identifiable and correctable underlying cause
  • Patients who are intolerant of rate-controlling medications
  • Patients with congestive heart failure - Arrhythmia can decompensate CHF, or cause CHF in an already sick heart
  • Patients < 65 years
67
Q

**

Which one of the following is not associated with increased serum troponin level?

A. Acute sepsis.
B. Renal failure.
C. Pulmonary embolism.
D. Aortic dissection.
E. Bradyarrhythmias.

A

E. Bradyarrhythmias.

There is a long list of conditions that can lead to elevated serum troponin levels in the absence of acute coronary syndrome and myocardial infarction. Bradyarrhythmias do not cause elevated serum troponin levels.

The causes for elevation of serum troponin levels are summarized in the following table.

68
Q

Which one of the following conditions would be associated with silent (painless) myocardial infarction?

A. Polycythemia vera.
B. Hypertension.
C. Diabetes.
D. Migraine.
E. Hypercholesterolemia.

A

C. Diabetes.

Typical presentation of myocardial infarction (MI) is with chest pain and discomfort that may radiate to the jaw, left arm or epigastrium. Nausea, vomiting and weakness are other features. Atypical presentation is with fatigue, weakness, syncope and sometimes shortness of breath.

Approximately one in three MIs are silent and the only presentation is changes on ECG in the absence of symptoms or presence of vague and non-specific symptoms. Silent MI is more common among the following patient groups:
* Women
* The elderly
* Diabetes

69
Q

A 76-year-old woman presents with complaint of palpitation. She is a known case of persistent atrial fibrillation for the past one year and is on rate control medications. She is not diabetic and has never had a stroke or TIA. She denies any episode of chest pain or shortness of breath. On examination, her BP is 130/85 mmHg with an irregular pulse of approximately 90 bpm. An ECG shows atrial fibrillation (AF). Which one of the following is the most appropriate management regarding prevention of stroke in this patient?

A. Digoxin.
B. Metoprolol.
C. Aspirin.
D. Warfarin.
E. Heparin.

A

D. Warfarin.

AF is manage either by rate control as the preferred method using beta blockers (first choice) or calcium channel blockers (diltiazem, verapamil), or rhythm control with flecainide, or amiodarone. Systemic thromboembolism is the most feared complication of AF, as one out of five patients with AF will develop cerebrovascular accidents (stroke or TIA) in one year. For this reason, the risk of stroke should be assessed and prevention considered for every patient with AF longer than 48 hours. CHA2DS2 -VASc scoring system is used for this objective in patients
with AF without underlying valvulopathies.

C: Congestive heart failure / Left ventricular dysfunction (1)
H: Hypertension (1)
A2: Age ≥75 years (2) 
D: Diabetes mellitus (1)
S2: Previous history of stroke or TIA (2)
V: Vascular disease i.e., peripheral arterial disease, previous MI, aortic plaques (1)
A: Age 65-74 years (1)
Sc: Sex category (i.e. female sex) (1)

NOTE - maximum score is 9 since age may contribute 0, 1, or 2 points 9

This woman is 76 years old (2 points), has no previous history of stroke or TIA (0) and is not diabetic (0) or hypertensive (0). There is no history of MI, peripheral arterial disease, or aortic plaque (0). She does not have congestive heart failure or left ventricle dysfunction (0). She receives one additional point for being female. Collectively, this woman has a CHA2DS2 - VASc score of 3; therefore, she should be started on warfarin as the most appropriate option. Heparin should also be started to counteract the initial prothrombotic effects of warfarin until INR is within the therapeutic range of 2-3.

NOTE - Although the cut-off point for type 1 hypertension is defined as a systolic blood pressure of 130 mmHg or higher by the American Heart Association (AHA), the cut-off value for hypertension in CHA2DS2 - VASc score remains 140mmHg or higher.

Option A: Digoxin is occasionally used for rate or rhythm control in AF. It does not prevent thromboembolic events.

Option B: Metoprolol is used for rate control not anticoagulation and thromboembolism prophylaxis.

Option C: Aspirin is used in patients with nonvalvular AF and a CHAD2S2 score of 0 or 1 (in carefully selected patients where anticoagulation outweighs the risk of bleeding).

Option D: Heparin is used initially in conjunction with warfarin and stopped once the INR is within the therapeutic range (2-3).

70
Q

According to current Australian Resuscitation Council Guidelines, which one of the following would form the basis for diagnosing an acute ST segment elevation myocardial infarction (STEMI)?

A. Chest pain with elevated troponin.
B. Chest pain/discomfort that often radiates, is provoked by exercise and relieved by rest.
C. Chest pain/discomfort suggestive acute myocardial infarction in the presence of ST segment elevation more than 1mm in two contagious leads, or newly developed left bundle branch block.
D. Chest pain/discomfort, ST depression or T-wave inversion, and elevated troponin.
E. Unprovoked sustained chest pain with 12-lead ECG that might be normal, with ST segment depression, or T-wave inversion.

A

C. Chest pain/discomfort suggestive acute myocardial infarction in the presence of ST segment elevation more than 1mm in two contagious leads, or newly developed left bundle branch block.

According to guidelines of Australian Resuscitation guideline (#14), the diagnosis of an STEMI is established if there are myocardial ischemia symptoms (non-specific in silent MI) and either or both of the following:
* ST segment elevation greater than one millimeter in two contagious leads in the presence of symptoms.
* Development of new left bundle branch block.

71
Q

A new systolic murmur is noted one week after an acute ST segment elevation myocardial infarction in a 65-year-old man. Echocardiography reveals an ejection fraction (EF) of 55%. Which one of the following is the most likely underlying cause for this murmur?

A. Papillary muscle rupture.
B. Papillary muscle dysfunction.
C. Mitral valve stenosis.
D. Aortic regurgitation.
E. Ventricular septal defect (VSD).

A

B. Papillary muscle dysfunction.

Post-myocardial infarction (MI) papillary muscle dysfunction (rupture included) is the most common and well-known complication of the left ventricle MI, resulting in mitral valve insufficiency and regurgitation.

Dysfunction of the papillary muscle is often associated with a normal or minimally decreased ejection fraction (EF), while the rupture of this muscle (option A) causes significant drop in EF. This patient has a normal EF.

Option C: Mitral valve stenosis is not a complication of MI.

Option D and E: Aortic valve regurgitation and VSD are not common complications in MI. Furthermore, these conditions are associated with decreased EF. This patient has a normal EF.

72
Q

Which one of the following is the most appropriate course of action for pulseless electrical activity?

A. Adrenaline 1mg intravenously and investigating for reversible causes.
B. Atropine up to 3 mg intravenously and looking for the reversible causes.
C. Start cardiopulmonary resuscitation, give 1mg adrenaline intravenously and look for reversible causes.
D. Defibrillation, CPR, defibrillation, CPR, and IV adrenaline.
E. No cardiopulmonary resuscitation as pulseless electrical activity in not likely to reverse.

A

C. Start cardiopulmonary resuscitation, give 1mg adrenaline intravenously and look for reversible causes.

Pulseless electrical activity (PEA) is managed by starting CPR (30:2) immediately, securing an airway and providing intravenous access. If not responsive, IV adrenaline should be given. Looking for a correctable cause of PEA and correcting that is always an important part of management.

Defibrillation has no role in management of PEA as the organized electrical activity of the heart is preserved.

There is no recommendation to support the use of atropine in PEA or asystole.

73
Q

A 67-year-old woman presents to your practice with complaint of dyspnea, especially on exertion. On heart auscultation, there is a systolic murmur heard over the apex, and a diastolic murmur in the left upper sternal border. The pulmonary component of S2 is split. Which one of the following is the most likely diagnosis?

A. Aortic valve stenosis.
B. Mitral valve regurgitation.
C. Pulmonary valve stenosis.
D. Tricuspid valve incompetence.
E. Mitral valve stenosis.

A

A. Aortic valve stenosis.

Typically, a split S2 is characteristic of either mitral valve regurgitation (MR) or aortic valve stenosis (AS) (may be seen in other cardiac conditions such as bundle branch block as well). The murmur of mitral regurgitation occurs during systole when the mitral valve fails to completely close and there is blood flow turbulence around the valve.

The typical murmur of the AS is an early systolic harsh murmur best heard over the right upper sternal border (in the second intercostal space) that can radiate to one or both carotid areas. However, in elderly patient the murmur may be best heard over the apex and resembles murmurs caused by MR.

On the other hand, calcified stenotic leaflets of the aortic valve may fail to tightly close during the diastole and give rise to valve regurgitation and a diastolic murmur.

With the history of exertional dyspnea, and the above mentioned murmurs and more importantly the split S2, AS will be the most likely diagnosis.

74
Q

A 63-year-old man visits you because of progressive shortness of breath brought on by exertion. He does not smoke and denies any chest pain, known cardiac disease, or hypertension. On examination, a systolic murmur over the left sternal border, a diastolic murmur over the left lower sternal border, and a systolic murmur over the apex are heard. Which one of the following is the most likely diagnosis?

A. Mitral valve stenosis.
B. Mitral valve regurgitation.
C. Aortic valve stenosis.
D. Aortic valve regurgitation.
E. Tricuspid valve regurgitation.

A

D. Aortic valve regurgitation.

Of all the valvular lesions, aortic valve regurgitation (AR) is capable of producing three different murmurs.

AR results in backflow of blood from the aorta into the left ventricle during diastole. This causes a diastolic decrescendo murmur. Since this murmur is caused by spillage of blood from the aorta into the left ventricle it is best heard over the left sternal border.

The backflow of blood from the aorta during the diastole results in increased end-diastolic left ventricular volume. The increased volume gives rise to a functional flow murmur during the systole which is often best heard between the apex and the left sternal border.

Additionally in severe AR, the backflow of the blood during the diastole causes a rumbling mid-diastolic jet murmur (Austin-Flint murmur). The jet flow strikes the anterior leaflet of the mitral valve and closes it prematurely.

Option A: Mitral valve stenosis is characterized by an opening snap after the S2 and diastolic rumbling murmur best heard in the apex.

Option B: Mitral valve regurgitation causes a holosystolic systolic murmur radiating to the axilla and best heard over the apex.

Option C: Aortic stenosis is associated with an ejection murmur in aortic area that may be associated with thrills. It has a harsh quality and may radiate to the carotids.

Option E: Tricuspid valve regurgitation is associated with a holosystolic murmur in some patients.

Aortic valve regurgitation (AR) causes blood to flow backward from the aorta into the left ventricle during diastole. This condition can produce three different murmurs:

  1. Diastolic Decrescendo Murmur:
    • Cause: Blood flows back into the left ventricle from the aorta during diastole.
    • Best Heard: Left sternal border.
  2. Systolic Flow Murmur:
    • Cause: Increased end-diastolic volume in the left ventricle leads to a functional flow murmur during systole.
    • Best Heard: Between the apex and the left sternal border.
  3. Austin-Flint Murmur:
    • Cause: In severe AR, the backflow of blood creates a mid-diastolic rumbling jet that hits the anterior leaflet of the mitral valve, causing it to close prematurely.
    • Description: Rumbling mid-diastolic murmur.
  • Mitral Valve Stenosis (Option A):
    • Murmur: Diastolic rumbling murmur with an opening snap after S2.
    • Best Heard: Apex.
  • Mitral Valve Regurgitation (Option B):
    • Murmur: Holosystolic murmur radiating to the axilla.
    • Best Heard: Apex.
  • Aortic Stenosis (Option C):
    • Murmur: Harsh ejection murmur in the aortic area, may be associated with thrills.
    • Radiation: To the carotids.
  • Tricuspid Valve Regurgitation (Option E):
    • Murmur: Holosystolic murmur in some patients.

This simplified comparison helps to identify and understand the different murmurs associated with various valvular lesions, making it easier to study and remember.

75
Q

A 67-year-old man presents to the emergency department with facial puffiness and swelling of the right arm and upper chest. He is moderately short of breath. On examination, his face, arm and upper chest are slightly cyanosed and puffy. The rest of the exam is inconclusive. Which one of the following would be the most appropriate next investigation?

A. Chest X-ray.
B. Echocardiography.
C. ECG.
D. RAST.
E. Angiography.

A

A. Chest X-ray.

The clinical picture is consistent with superior vena cava (SVC) syndrome. SVC syndrome results from any condition that leads to obstruction of blood flow through the SVC. Obstruction can be caused by external compression of the SVC by adjacent pathologic processes involving the right lung, lymph nodes, and other mediastinal structures, or by thrombosis within the SVC. In some cases, both external compression and thrombosis coexist.

Conditions that can lead to SVC syndrome can be malignant or non-malignant:

Malignant causes - malignant mediastinal tumors are the most common cause (>80%). Of malignant tumors bronchogenic carcinoma accounts for 75-80% of cases, with most of these being small cell carcinomas. Non-Hodgkin lymphoma is responsible for 10-15% of mediastinal malignancies resulting in SVC syndrome. Rare malignant causes include Hodgkin lymphoma, metastatic cancers, primary leiomyosarcoma of the mediastinal vessels and plasmocytomas.

Non-malignant causes - some of these causes are:
* Mediastinal fibrosis
* Vascular diseases, such as aortic aneurysm, vasculitis, and arteriorvenous fistulas
* Infections, such as histoplasmosis, tuberculosis, syphilis, and actinomycosis
* Benign mediastinal tumors such as teratoma, cystic hygroma, thymoma, and dermoid cyst
* Cardiac causes, such as pericarditis and atrial myxoma
* Thrombosis related to the presence of central vein catheters

Early in the clinical course of SVC syndrome, partial obstruction of SVC may be asymptomatic, or the symptoms are so minor that are overlooked, but as it advances toward complete SVC obstruction, the classic symptoms and signs become more obvious:
* Dyspnea - the most common symptom (63% of patients)
* Facial swelling
* Head fullness
* Cough
* Arm swelling
* Chest pain
* Dysphagia
* Orthopnea
* Distorted vision
* Hoarseness
* Stridor
* Headache
* Nasal stuffiness
* Nausea
* Pleural effusion
* Lightheadedness

The characteristic physical findings include venous distention of the neck and chest wall, facial edema, upper extremity edema, mental status changes, plethora, cyanosis, papilledema, stupor and even coma. Bending forward or lying down can aggravate the signs and symptoms.

Since most cases of SVC are due to mediastinal malignancies, a chest X-ray is always the most appropriate initial investigation. Chest X-ray may reveal a widened mediastinum or a mass in the right side of the chest. One study showed that only 16% of patients with SVC syndrome had a normal chest X-ray.

CT scan, MRI, and angiography (option E) can be use for more detailed evaluation, but not as initial assessment.

RAST (option D) stands for radioallergosorbent test and is a blood test for finding the allergen a patient is allergic to, and has no role in diagnosis of SVC syndrome.

Option B and C: ECG and echocardiography are not diagnostic for SVC.

76
Q

A 45-year-old man presents to your clinic with hand and arm swelling and pain which developed a few hours after working with a chainsaw. He has no family or personal history of thrombophilia or deep vein thrombosis. On physical exam, he is in good health otherwise. Generalized non-pitting edema of the right hand and arm up to the shoulder is noted. The arm is slightly cyanosed, but not warm to touch. The remainder of the exam is inconclusive. Which one of the following is the most likely diagnosis?

A. Lymphedema.
B. Subclavian vein thrombosis.
C. Brachial plexus injury.
D. Cellulitis.
E. Lymphangitis.

A

B. Subclavian vein thrombosis.

The clinical picture is suggestive of upper extremity deep vein thrombosis (UEDVT) (thrombosis of axillary or subclavian vein). UEDVT is rare but increasing in incidence with considerable morbidity. Pulmonary embolism may occur in as many as one-third of the patients.

The increased incidence of UEDVT in the recent decades is directly related to increasing use of central catheterization for chemotherapy, bone marrow transplantation, dialysis and parenteral nutrition.

UEDVT has been reported in 25% of those with central catheters.

UEDVT is classified as primary or secondary on the basis of pathogenesis:

PRIMARY UEDVT
Primary UEDVT is a rare disorder (2 per 100 000 persons per year) which is subdivided into two other categories:

Effort thrombosis (the so-called Paget-Schroetter Syndrome) - Patients with Paget-Schroetter Syndrome develop spontaneous UEDVT, usually in their dominant arm, after strenuous activity such as rowing, wrestling, weight lifting, or baseball pitching, but are otherwise young and healthy. The heavy exertion causes microtrauma to the vessel intima and leads to activation of the coagulation cascade. Significant thrombosis may occur with repeated insults to the vein wall, especially if mechanical compression of the vessel is also present.

Idiopathic UEDVT- Patients with idiopathic UEDVT have no known trigger or obvious underlying disease. Idiopathic UEDVT can be, however, associated with occult cancer. In one study, one fourth of patients presenting with idiopathic UEDVT were diagnosed with cancer (most commonly lung cancer or lymphomas) within one year of follow-up.

Thoracic outlet obstruction refers to compression of the neurovascular bundle (brachial plexus, subclavian artery, and subclavian vein) as it exits the thoracic inlet. Although this disorder may initially cause intermittent, positional extrinsic vein compression, repeated trauma to the vessel can result in dense, perivascular, fibrous scar tissue formation that will compress the vein persistently. Compression of the subclavian vein typically develops in young athletes with hypertrophied muscles who do heavy lifting or completely abduct their arms. Cervical ribs, long transverse processes of the cervical spine, musculofascial bands, and clavicular or first rib anomalies are sometimes found in these patients. Therefore, cervical spine and chest plain films should be obtained in all patients undergoing evaluation for thoracic outlet syndrome.

Regardless of the etiology, UEDVT can be associated with the following complications:
* Pulmonary embolism (in one-third of patients)
* Persistent upper extremity pain and swelling
* Superior vena cava syndrome
* Loss of vascular access.

SECONDARY UEDVT
Secondary UEDVT develops in patients with central venous catheters, pacemakers, or cancer and accounts for most cases of UEDVT. Catheter-related thrombosis is caused by several factors including damage to the vessel wall during catheter insertion or infusion of medication, or the catheter impeding the blood flow through the vessel and stasis.

Regardless of the etiology, UEDVT can have the following clinical features:
* Vague shoulder or neck discomfort
* Pain and discomfort of the arm
* Non-pitting edema of the hand and arm
* Extremity cyanosis
* Dilated cutaneous veins
* Supraclavicular fullness
* Fever (often low-grade) – higher fevers may be caused by thrombophlebitis or the underlying malignancy
* Facial edema and cyanosis if superior vena cava syndrome has occurred

If the thoracic outlet syndrome is the etiology of the UEDVT the additional following findings are other possible findings:
* Brachial plexus tenderness
* Pain radiating to the arm and forearm
* Arm or hand atrophy
* Hand weakness
* Referred pain from the brachial plexus to the fourth and fifth fingers

NOTE (1) - At occasions, UEDVT may be asymptomatic.

NOTE (2) - Thrombophilia seems to be associated with increased risk of UEDVT.

Option A: Lymphedema can be a differential diagnosis, but there is no predisposing factor in history favoring this diagnosis.

Option C: With brachial plexus injury, neurologic symptoms would have been present. However, UEDVT caused by thoracic outlet syndrome may be associated with concurrent neurologic dysfunctions of the brachial plexus.

Option D and E: Lymphangitis and cellulitis are often associated with more systemic symptoms e.g. fever. The patient may look ill. Erythema and warmth of the limb are other features that are absent here.

77
Q

Worried parents of a 4-year-old boy bring him to the Emergency Department because he is breathless and his heart is racing. An ECG strip is obtained and is as follows. Which one of the following is the most appropriate immediate management of this child?

A. Reassure the parents as it is sinus tachycardia.
B. Intravenous adenosine.
C. Intravenous lignocaine.
D. Intravenous beta blockers.
E. Immerse the child’s face in cold water.

A

B. Intravenous adenosine.

Condition:
- Non-sustained wide QRS complex tachycardia: Episodes of fast heartbeats with wide QRS complexes (>0.09 seconds) that start and stop on their own.
- Can be ventricular tachycardia (VT) or supraventricular tachycardia (SVT) with aberrancy.
- In children, it’s often SVT with aberrancy, but treat as VT until proven otherwise.

When to Treat:
- Asymptomatic: Identify and correct the underlying cause.
- Symptomatic or sustained: Requires treatment.

Treatment Based on Hemodynamic Status:

  1. Hemodynamically Unstable (e.g., hypotension, altered mental status, signs of shock):
    • If there’s a pulse: Electrical cardioversion.
    • If no pulse: Defibrillation.
  2. Hemodynamically Stable (e.g., no hypotension, no altered mental state, no shock):
    • In adults: Use an antiarrhythmic like amiodarone, lidocaine, or sotalol.
    • In children: Start with intravenous adenosine if the rhythm is regular and QRS complexes are monomorphic, as it’s often SVT with aberrancy.

Why Adenosine for Children:
- Adenosine is effective for SVT with aberrancy.
- If adenosine doesn’t work, consider VT and use amiodarone or procainamide after consulting a specialist.

Options Explained:

  • Option A (Reassurance): Not suitable. This child needs treatment and investigation.
  • Option C (Amiodarone/Lidocaine): Use only if adenosine fails.
  • Option D (Beta Blockers): Used orally for mild symptoms in an outpatient setting.
  • Option E (Vagal Maneuvers): Effective for narrow QRS tachycardia, not wide QRS.

Summary:
- First-line treatment for symptomatic non-sustained wide QRS tachycardia in children: Intravenous adenosine.

The ECG shows episodes of tachycardia with wide monomorphic QRS complexes that do not sustain and terminate spontaneously. This is called non-sustained wide QRS complex tachycardia. A wide complex for this matter is defined as a QRS of >0.09 second (90 msec). Wide QRS complex tachycardia is either due to ventricular tachycardia or a supraventricular tachycardia (SVT) with aberrancy. While most wide QRS complexes in adults originate from ventricles, in children they often represent SVT with aberrancy. However, due to the potential progression of ventricular tachycardia (VT) to ventricular fibrillation and cardiac arrest, any wide QRS complex tachycardia should be considered VT and managed accordingly until proven otherwise.

Asymptomatic non-sustained wide QRS complex tachycardia does not often require treatment other than identifying and correction of the underlying cause. Symptomatic non-sustained wide QRS complex tachycardia and sustained wide QRS complex tachycardias whether symptomatic or asymptomatic require treatment.

Treatment options depend on the patient’s hemodynamic status:
* In patients with wide QRS complex tachycardia who are hemodynamically unstable, represented by hypotension, acutely altered mental status or signs of shock, the treatment is with electrical cardioversion if there is a pulse. Defibrillation is the choice in the absence of a pulse.

  • In adult patients with sustained wide QRS who are asymptomatic or have symptoms other than those mentioned above, or non- sustained wide QRS complex tachycardia with symptoms other than those mentioned above, pharmacotherapy with an antiarrhythmic medication is the treatment of choice. Amiodarone is first-line. Lignocaine or sotalol can be used alternatively.
  • In children with sustained wide QRS complex tachycardia who are asymptomatic, or have symptoms but are hemodynamically stable (no hypotension, no acutely altered mental state, no signs of shock), or in children who have non-sustained wide QRS complex tachycardia with symptoms other than those related to hemodynamic instability, it is recommended that treatment start with adenosine if the rhythm is regular and QRS complexes are monomorphic. The rationale behind this is the fact that such tachycardias are often SVTs with aberrancy; hence, responsive to adenosine. However, if there is no response to adenosine, VT should be considered and treated with amiodarone or procainamide after expert consultation or under expert supervision.

This child has symptomatic non-sustained wide QRS complex tachycardia but his symptoms are not any of those representing hemodynamic instability; so, the most initial treatment to consider for him must be intravenous adenosine.

Option A: This child has symptomatic non-sustained wide complex tachycardia, probably with an underlying pathology. His parents cannot be reassured, and this child’s tachycardia requires treatment as well as investigations for an underlying cause.

Option C: Lignocaine, amiodarone, procainamide, etc. will be considered after adenosine proved ineffective to control the tachycardia, and the wide QRS complex tachycardia is likely to be VT. These medications should be used after consultation or under supervision of an expert.

NOTE - of these medications, amdiodarone is the safest to use.

Option D: Oral (not intravenous) beta blockers are used as first-line in treatment of non-sustained wide QRS complex tachycardias in outpatient setting if the symptoms are mild or not representative of hemodynamic instability.

Option E: Vagal stimulation by maneuvers such as immersion of face in cold water, Valsalva maneuver, carotid massage etc. are used as the initial management for patients with *narrow QRS complex *tachycardia because such tachycardias are not likely to have ventricular origin. This child has wide QRS.

78
Q

A 72-year-old man presents to the Emergency Department with severe retrosternal chest pain and shortness of breath. ECG reveals ST segment elevation myocardial infarction (STEMI). Angiography is performed which shows 70% stenosis of the right coronary artery and 30% of the left. Which one of the following would be the most appropriate management option?

A. Pacemaker.
B. Thrombolytic therapy.
C. Angioplasty of the right coronary artery.
D. Angioplasty of the left coronary artery.
E. Angioplasty of both right and left coronary arteries.

A

C. Angioplasty of the right coronary artery.

Timely angioplasty is the gold-standard treatment of STEMI. Angioplasty commonly includes passing an arterial catheter from the femoral or radial artery into the aorta and from there into the coronary arteries. Contrast material is then injected to visualize a map of coronary arteries and possible lesions.

Significant stenosis (>50%) may be considered for ballooning, atherectomy, stenting, etc. Insignificant coronary artery disease is defined as a stenosis of less than 50% (consensus-based). Insignificant stenosis does not often require intervention.

In this patient, only the right coronary artery (70% stenotic) requires angioplastic intervention.

Option A: Pacemaker insertion is not a treatment option for myocardial infarction; however, it might be used for treatment of heart block as a complication of MI at particular occasion.

Option B: Thrombolytic therapy was the correct option if angioplasty would not be available in a timely fashion.

Option D and E: The left coronary artery does not require intervention for now.

79
Q

A 56-year-old Aboriginal Australian man is brought to the Emergency Department of a local hospital with severe chest pain radiating to the left arm, hypotension, and bradycardia. An ECG confirms ST elevation in leads I, aVL, V5 and V6. Which one of the following fibrinolytic agents cannot be used for treatment of this patient?

A. Alteplase.
B. Tenecteplase.
C. Urokinase.
D. Streptokinase.
E. Reteplase.

A

D. Streptokinase.

There is a high prevalence of IgG anti-streptokinase antibody among Aboriginal Australian patients, making this drug an inappropriate option for thrombolytic therapy in acute ST elevation myocardial infarction in this patient group. This antibody makes treatment with this agent ineffective.

80
Q

A 71-year-old man with a history of congestive heart failure for the past 10 years, controlled on perindopril and hydrochlorothiazide, presents to the Emergency Department with a one-day history of palpitation. He has a blood pressure of 135/90 mmHg, an irregular pulse rate of 98 bpm, and a respiratory rate of 18 breaths per minute. Lungs are clear to auscultation. No ankle edema or raised jugular venous pressure is noted. An ECG is obtained and is shown in the following photograph. Which of the following would be the next best step in management?

A. Aspirin.
B. Metoprolol.
C. Warfarin.
D. Digoxin.
E. Diltiazem.

A

B. Metoprolol.

The ECG is characteristic of atrial fibrillation (AF). The most appropriate initial management of AF in a patient with stable hemodynamic status is rate control. Rate control often leads to an improvement in symptoms in patients with heart failure. In addition, slowing of the ventricular rate often leads to moderate or, in some cases, marked improvement in left ventricular function.

Cardioselective beta-blockers (metoprolol, atenolol, and carvedilol) are first-line medications for this purpose. Anticoagulation should be considered if AF persists beyond 48 hours or if the duration is unknown. Digoxin should be added if a second agent is necessary to achieve adequate rate control. However, the efficacy of digoxin decreases with physical activity.

If rate control with either beta-blockers or a combination of beta-blockers and digoxin has not been achieved, amiodarone may be useful either alone or in combination with other rate-slowing agents.

It should be noted that in patients with decompensated heart failure the initiation or increase of beta blockers is contraindicated. In such patients, the use of digoxin is suggested. However, digoxin is often ineffective if used alone, especially in patients with an elevated sympathetic tone. Also, for patients with acute AF with the rapid ventricular response associated with heart failure, amiodarone may be helpful to slow the ventricular rate. Although usually used for rhythm control, amiodarone can also provide a degree of rate control while it is being loaded and thereafter. Amiodarone is not recommended as a chronic rate-control medication, but in the acute setting can assist with rate control as it is being loaded, or can be used as a temporary rate-control medication in a patient who is unable to tolerate other therapies.

Even though this patient has heart failure, since the heart failure is rather stable and not decompensated, beta-blockers (metoprolol) are the most appropriate next step in management.

NOTE - Acute decompensated heart failure (ADHF) can be defined as the sudden or gradual onset of the sign and/or symptoms of heart failure requiring unplanned office visits, emergency department visits, or hospitalization. Regardless of the underlying precipitant of the exacerbation, pulmonary and systemic congestion due to increased left- and right-heart filling pressure is nearly a universal finding in ADHF.

Acute decompensated heart failure (ADHF) may present with:
* Pulmonary edema
* Cardiogenic shock
* Weight gain and fluid retention
* Exertional dyspnea
* Orthopnea
* Raised jugular venous pressure
* Peripheral edema

Option A: Aspirin is used for patients in whom AF is not caused by a valvular lesion and has a CHA2DS2 - VASc score of zero (or 1 in selected cases).

Option C: Anticoagulation with warfarin (after reaching an INR of 2-3 by initial administration of heparin) is considered the second most important step in whom AF has lasted over 48 hours or is of unknown duration. With an AF of only one-day duration anticoagulation is not required for now.

Option D: Digoxin could have been an appropriate option if the patient’s heart failure was decompensated and beta blockers were contraindicated.

Option E: Although non-dihydropyridine calcium channel blockers (verapamil and diltiazem) are second-line medications for rate control in patients with AF, extreme caution should be considered in patients with heart failure, due to the risk of heart failure exacerbation and decompensation.

81
Q

A 72-year-old woman is brought to the Emergency Department with chest pain of several hours duration. Of physical findings on examination, a blood pressure of 98/50 mmHg and an irregular pulse of 120 bpm are remarkable. An ECG shows ST elevation in leads V2 and V4. Which one of the following will be the most appropriate initial management option?

A. Cardioversion.
B. Thrombolysis with TPA.
C. Lidocaine.
D. Verapamil.
E. Digoxin.

A

B. Thrombolysis with TPA.

Chest pain and ST elevation in leads V2 through V4 suggests ST-segment elevation myocardial infarction (STEMI) of the anterior wall. On the other hand, an irregular pulse of 120 bpm suggests atrial fibrillation (AF). AF is seen in 10-15% of the patients with acute myocardial infarction (MI) as a complication.

The onset of AF in the first hours of MI is usually caused by left ventricular failure, ischemic injury to the atria, or right ventricular infarction. Pericarditis and all conditions leading to elevated left atrial pressure can also lead to AF in association with an MI. The presence of AF during an MI is associated with an increased risk of mortality and stroke, particularly in patients who have anterior wall MI.

Although both conditions are simultaneously present in this patient, urgent management of the MI takes precedence over that of AF. If the patient presents within the first 12 hours of symptoms onset, reperfusion therapy either by percutaneous coronary intervention (PCI) or thrombolytic therapy with thrombolytic medications such as tissue plasminogen activator (TPA), reteplase, alteplase, tenecteplase, etc. should be considered as the most appropriate next step in management. PCI (not an option), if performed in a timely fashion and by an experienced interventional cardiologist, has better outcomes and fewer complications compared to thrombolytic therapy.

AF persisting after reperfusion therapy should managed accordingly:

Immediate electrical cardioversion is indicated for all patients who are hemodynamically unstable, such as those with new or worsening ischemic pain and/or hypotension. Synchronized electrical cardioversion to treat atrial fibrillation begins with 200 J (or the biphasic equivalent). Conscious sedation (preferred) or general anesthesia is advisable prior to cardioversion.

For patients who do not develop hypotension, a beta-blocker can be used. For example, metoprolol may be given in 5-mg intravenous boluses every 5-10 min, with a maximum dose of 15 mg. Intravenous diltiazem is an alternative for slowing the ventricular rate, but it should be used with caution in patients with moderate-to-severe heart failure. In patients with new-onset sustained tachycardia (absent before MI), conversion to sinus rhythm should be considered as an option.

Atrial fibrillation and atrial flutter confer an increased risk of thromboembolism; therefore, anticoagulation with either unfractionated heparin or low molecular weight heparin (LMWH) should be started if contraindications are absent.

82
Q

Which one of the following is the most appropriate treatment for venous ulcers of lower limb?

A. Bed rest with limb elevation.
B. Antibiotics.
C. Tissue debridment.
D. Compression stocking and a walking program.
E. Aspirin and statins.

A

D. Compression stocking and a walking program.

Venous hypertension is associated with morphologic changes in the capillary and lymphatic microcirculation, resulting in important physiologic changes such as capillary leak, fibrin deposition, erythrocyte and leukocyte sequestration, thrombocytosis and inflammation. Collectively, these processes impair oxygenation of the skin and subcutaneous tissue. Severe venous hypertension and hypoxia leads to edema, subcutaneous fibrosis, hyperpigmentation and ulcer formation.

Continuous compression therapy, typically achieved with gradual compression stockings or compression bandage is a very important and essential part of treatment of chronic venous disease with or without ulceration.

The mechanism by which compression therapy produces beneficial effects is not yet understood; however, the following have been suggested:
* improvement in venous hemodynamics by reducing venous reflux increasing deep venous flow velocity
* improvement in lymphatic flow and cutaneous microcirculation decreasing ambulatory venous pressure
* alteration in certain pro-inflammatory matrix metalloproteinases present in chronic ulcers that result in favorable changes promoting ulcer healing
* compression that is intermittent enhances fibrinolysis, a potentially important mechanism in reducing fibrosis and enhancing ulcer healing

Compression therapy is contraindicated in patients with moderate to severe peripheral artery disease (PAD), and should be used with extreme caution in patients with mild to moderate PAD, evident by an ankle-brachial index (ABI) of < 0.9 but > 0.6.

In the presence of arterial disease, compressiom stocking can cause skin necrosis that in few instances may lead to amputation. Active cellulitis is another contraindication for compression therapy.

83
Q

A 57-year-old man presents to the Emergency Department with sudden onset of severe pain and pallor in the left lower limb. He is a known case of ischemic heart disease (IHD) and peripheral vascular disease (PVD). On examination, left femoral pulse is felt, but the left popliteal pulse is absent. Which one of the following is the most appropriate next step in management?

A. Intravenous unfractionated heparin (UFH).
B. Low molecular weight heparin (LMWH).
C. Warfarin.
D. Verapamil.
E. Embolectomy.

A

A. Intravenous unfractionated heparin (UFH).

Scenario:
- Sudden onset of pain and pallor in a limb
- Absence of popliteal pulse

Diagnosis:
- Acute limb ischemia

Immediate Treatment:
- Intravenous bolus dose of unfractionated heparin (UFH):
- Prevents further thrombus formation and distal thrombosis
- Administer 5000 IU loading dose, followed by 1250 IU/hour
- Adjust dosage based on active partial thromboplastin time (APTT)

Why UFH?
- Quick action and easy to reverse with protamine sulfate
- Suitable for patients who might need surgery or other invasive procedures

Other Options:
- Option B: Low Molecular Weight Heparin (LMWH):
- Effective but not ideal if surgery might be needed due to difficulty reversing its effects

  • Option C: Warfarin:
    • Oral anticoagulant, not suitable for immediate management due to initial pro-thrombotic risk
    • Used for long-term management
  • Option D: Verapamil:
    • Calcium channel blocker, no benefit in acute limb ischemia
  • Option E: Embolectomy:
    • Possible definitive treatment, but heparin must be given first to stabilize the patient

Summary:
- Start with UFH immediately to prevent further clotting and stabilize the patient before deciding on further treatments like surgery or thrombolytic therapy.

Sudden onset of pain and pallor suggests acute limb ischemia. Absence of popliteal pulse confirms the diagnosis with high certainty. In approach to acute limb ischemia, administration of bolus dose of unfractionated heparin (UFH), intravenously, is the most appropriate next step in management once the diagnosis of acute arterial occlusion has been made by history and physical examination.

Anticoagulation prevents further propagation of thrombus and inhibits thrombosis distally in the arterial and venous systems due to low flow and stasis. Administration of heparin should not be delayed while waiting for diagnostic procedures to be performed. A loading dose of 5000 IU is given, which is then followed by 1250IU/hour. Further adjustment is guided by active partial thrombin time (APTT).

Option B: LMWH can be uses with the same efficacy of UFH, except in unstable patient or where a surgical are other invasive procedure
may follow, in which case UFH must be used instead because the anticoagulation effect of UFH can be readily reversed by protamine sulfate. This patient will require either vascular catheterization and embolectomy, or thrombolytic therapy or, in worst case scenario, amputation of his leg. This makes UFH the most appropriate option.

Option C: Warfarin is an oral antagonist of vitamin K. In early stages after initiation, it is associated with increased risk of clot formation (pro- thrombotic). Warfarin is indicated for long-term management of this patient.

Option D: Verapamil is a non-dihydropyridine calcium channel blocker (CCB). Its use has no therapeutic benefit in patients with acute limb ischemia.

Option E: Embolectomy is one of the options for definite treatment of this patient, but heparin, as the crucial initial management, must be started before decision as to definite treatment is made.

84
Q

A 65-year-old woman presents to the Emergency Department after she collapsed at work. She was walking to her desk when she suddenly lost consciousness and fell down. According to her colleague, who is accompanying her, she was ‘out’ for only a few seconds, and regained consciousness shortly. One examination, she is overweight, has a blood pressure of 110/75 mmHg, pulse rate of 110 bpm and respiratory rate of 18 breaths per minute. You are arranging for an ECG when she collapses again. On a quick review, she is found to have a blood pressure of 80/45 mmHg. You feel no radial or carotid pulse. An ECG is obtained which is shown in the following photograph. Which one of the following is the most appropriate next step in management?

A. Intravenous bolus of normal saline.
B. Intravenous amiodarone.
C. Synchronized cardioversion.
D. Cardiopulmonary resuscitation and defibrillation.
E. Cardiac catheter ablation.

A

D. Cardiopulmonary resuscitation and defibrillation.

The ECG is typical for ventricular tachycardia (VT). In each lead, QRS complex are of same morphology; therefore, this patient has a monomorphic VT.

Approach to sustained VT (VT lasting more than 30 seconds) depends on the patient’s hemodynamic status. Those with hemodynamic instability are in emergency need for reversion of the rhythm to a sinus rhythm. In the presence of a palpable pulse, the initial management is by synchronized cardioversion if a pulse is palpable, and defibrillation (unsynchronized) if no pulse is felt.

Since this patient is pulseless, CPR should be immediately started and defibrillation performed when available and ready.

In patients with a shockable rhythm (ventricular fibrillation and pulseless VT), the first action is start chest compression while waiting for defibrillator to be ready. First defibrillation should be followed by 2 minutes of CPR. After the second defibrillation, adrenaline should be started and repeated every second cycle.

Hemodynamic instability is defined as the presence of any of the following:
* Chest pain
* Hypotension (systolic blood pressure < 90mmHg)
* Perfusion-related confusion or unconsciousness
* Unconsciousness
* Heart failure

Option A: In a patient with VT, the cause of hypotension is decreased cardiac output as result of pump failure not hypovolemia; therefore, intravenous fluids are useless.

Option B: Antiarrhythmic medications such as amiodarone are indicated as first-line management option in patients who are hemodynamically stable despite presence of VT, or for sustaining a sinus rhythm after treatment with cardioversion or defibrillation.

Option C: Synchronized cardioversion was initial management and the correct option if the patient had a palpable pulse.

Option E: Cardiac catheter ablation is the definite treatment in patients with regular supraventricular tachyarrhythmias if the cause is found to be an accessory pathway bypassing the normal cardiac conductive system e.g. Wolff-Parkinson-White syndrome.

85
Q

A 60-year-old man presents to the Emergency Department with complaint of palpitation for the past 2 weeks. On examination, he has blood pressure of 125/95 mmHg, and an irregular heart rate of 160 bpm. An ECG shows atrial fibrillation (AF). He is started on metoprolol for rate control, and electrical cardioversion is planned. Which one the following should be used for anticoagulation?

A. Heparin, 24 hours before the procedure.
B. Warfarin 4 weeks before, to 4 weeks after the procedure.
C. Aspirin.
D. Apixaban from 48 hours before, to 48 hours after the procedure.
E. Flecainide.

A

B. Warfarin 4 weeks before, to 4 weeks after the procedure.

Converting AF to sinus rhythm is associated with a small risk of thromboembolic events. The risk can be due to either pre-existing thrombus (more common) or de novo thrombus formation.

Embolization after conversion to sinus rhythm is often caused by the dislodgement of left atrial (LA) thrombi that has been formed during atrial stagnation; however, in a minority of patients the clot forms at the time of cardioversion.

This risk is increased in the immediate post-cardioversion period, whether planned or spontaneous. Patients undergoing cardioversion of AF of more than 48 hours duration represent a high-risk group, with an embolic risk of 1-5% in the first month if prophylactic anticoagulation is not given.

The 1-month risk of post-cardioversion thromboembolism can be decreased to less than 1% with anticoagulation starting from 1 month (4 weeks) before to 1 month (4 weeks) after the procedure. Oral anticoagulation with warfarin with a target INR of 2-3 is the most common method used.

Heparin is used initially before, or at the same time with warfarin to counteract the prothrombotic effects of warfarin early in the course of use, and is stopped once INR is within the therapeutic range of 2-3.

Option A: Heparin is also used for acute anticoagulation if emergency cardioversion has been considered, such as in patients with hemodynamic compromise due to AF, or who have failed to respond to rate control medications and are symptomatic.

Option C: Aspirin is unlikely to provide adequate prophylaxis against thromboembolism prior to and after cardioversion.

Option D: Apixaban and rivaroxaban are oral factor Xa inhibitor coagulants. Rivaroxaban has shown equal efficacy to warfarin. Apixaban is suggested to be superior to warfarin; however, their efficacy and safety prior to and immediately after cardioversion has not been adequately studied.

Option E: Flecainide is an antiarrhythmic medication used for pharmacological cardioversion with no effectiveness in preventing thromboembolism.

86
Q

A 65-year-old female patient undergoes percutaneous coronary intervention and stent placement through femoral artery, and is started on aspirin andclopidogrel. After 24 hours, she develops a pulsatile painful mass in the groin though which the catheter was sent in. Which one of the following is the definitive treatment of this mass?

A. Massage and application of compression.
B. Surgical repair.
C. Vitamin K.
D. Angiography.
E. Injection of thrombin into the mass.

A

E. Injection of thrombin into the mass.

The scenario is a typical description of a pseudoaneurysm as a complication of femoral artery catheterization.

A pseudo aneurysm is a hematoma that forms as the result of a leaking hole in an artery. The hematoma forms outside the arterial wall and is contained by the surrounding fibromuscular tissue. The hematoma must continue to communicate with the artery to be considered a pseudoaneurysm.

Pseudoaneurysm occurs in up to 7.5% of femoral artery catheterizations and can cause distal embolization, extrinsic compression on the neurovascular structures, rupture, and hemorrhage.

A pseudoaneurysm presents with a painful pulsatile groin mass. A bruit over the mass may or may not be heard. Duplex Doppler ultrasound may provide evidence of extra-arterial flow or there may be classic ‘to-and-fro’ Doppler waveform in the neck of the pseudoaneurysm.

Ultrasound-guided thrombin injection for the pseudoaneurysms of the iliac, femoral and peroneal arteries is safe, effective, and associated with few complications. It has emerged as the preferred treatment modality for pseudoaneurysms occurring as a result of percutaneous femoral arterial interventions (success rate: 97%).

The procedure should be performed by a physician and an ultrasonographer to enable continuous visualization of the pseudoaneurysm. Thrombin is injected into the sac of the pseudoaneurysm away from the neck under direct ultrasound guidance.

Option A: The ultrasound-guided compression is successful in 90% of case and was the treatment of choice previously; however, this approach is not favorable anymore because it needs prolonged compression time (up to 120 minutes), makes the patient uncomfortable, is associated with early recurrence and has limited success in treating large pseudoaneurysms.

Option B: Urgent surgical exploration is indicated for a threatened limb and when a percutaneous approach is not feasible. This method was the treatment of choice before 1985.

Option C: Vitamin K is not a treatment option. Any options suggesting cessation of clopidogrel is incorrect as this is associated with high mortality rate in a patient who has just undergone angioplasty. Moreover, it does not treat the aneurysm.

Option D: Angiographic intervention via a retrograde approach from the contralateral common femoral artery is only indicated if acute vessel occlusion due to distal emboli occurs. The event presents with the patient complaining of pain, pallor, parenthesis or decreased movement in the respective limb. Clinical examination may reveal a cold ischemic limb with absent pulses, an ankle-brachial index (ABI) < 0.5, or absent color flow and Doppler waveform on duplex ultrasound in the index artery.

Other Treatment Options:

•	A: Ultrasound-guided compression:
•	Successful in 90% of cases
•	Requires prolonged compression time (up to 120 minutes)
•	Can be uncomfortable for the patient
•	Higher risk of early recurrence and less effective for large pseudoaneurysms
•	B: Urgent surgical exploration:
•	Indicated for a threatened limb or when percutaneous approach is not feasible
•	Was the primary treatment before 1985
•	C: Vitamin K:
•	Not a treatment for pseudoaneurysm
•	Stopping clopidogrel is not recommended as it increases mortality after angioplasty
•	D: Angiographic intervention:
•	Indicated for acute vessel occlusion due to distal emboli
•	Presents with pain, pallor, paresthesia, or decreased limb movement
•	Examination may reveal cold ischemic limb with absent pulses and low ABI (< 0.5)
87
Q

A 30-year-old school teacher presents to the Emergency Department after she collapsed at school during exercise. She was unconscious for few seconds with full recovery afterwards. She now complains of shortness breath but denies chest pain. On examination, a systolic murmur is heard over the right second intercostal space radiating to the apex. Which one of the following could be the most likely diagnosis?

A. Patent ductus arteriosus.
B. Aortic stenosis.
C. Hypertrophic obstructive cardiomyopathy.
D. Ventricular septal defect.
E. Mitral stenosis.

A

B. Aortic stenosis.

Exercise-induced collapse and a murmur on examination are most likely to be caused by a structural heart disease such as valvular lesions or hypertrophic cardiomyopathies. Arrhythmias can also be a cause of exercise-induced hemodynamic collapse; however, arrhythmias usually do not give rise to murmurs.

This patient has a systolic murmur best heard over the right second intercostal space radiating to the apex. Of the options, only aortic stenosis (AS) is capable of producing such murmur.

The murmur associated with AS is a systolic ejection murmur. It is typically heard best at the base of the heart in the ‘aortic’ area that is the right second intercostal space. The murmur has a harsh quality and can be transmitted equally to the carotid arteries. The murmur may also radiate to the apex of the heart where it may have a different quality. It is also common for patients with AS to have mild mitral regurgitation with a systolic murmur best heard at the apex.

Option A: The murmur in patent ductus arteriosus (PDA) is heard in both systole and diastole (Gibson’s murmur or machinery murmur) with greatest intensity in the left infraclavicular region. Furthermore, PDA is congenital heart disease and very unlikely to have presented this late.

Option C: Hypertrophic obstructive cardiomyopathy (HOCM) can cause exertional syncope; however, characteristics of the murmur described in the scenario make HOCM less likely. Patients with HOCM may develop several types of systolic murmurs, but the two most common are related to left ventricular outflow tract (LVOT) obstruction and mitral regurgitation.

The LVOT obstruction murmur is a harsh crescendo-decrescendo murmur best heard over the left sternal border that can radiate to the axilla but not the neck.

Papillary muscle and/or chordae tendineae abnormalities associate with HOCM results in mitral valve regurgitation that can produce a mid- or late-systolic murmur that is best heard at the apex.

Option D: Ventricular septal defect is associated with a systolic murmur with greatest intensity in the left sternal border.

Option E: Severe mitral stenosis can cause syncope during exercise but the murmur is diastolic with the greatest intensity in the apex.

88
Q

A 54-year-old man presents to the emergency department with complaint of retrosternal chest pain that worse on inspiration and lying back and relieved by bending forward. On examination, he has a blood pressure of 138/97mmHg, pulse rate of 110bpm, shallow respiration at a rate of 26 breaths per minute and a temperature of 38.5°C. An ECG is obtained which is shown in the following photograph. Which one of the following is the most appropriate next step in management after administration of supplemental oxygen?

A. Non-steroidal inflammatory drugs (NSAIDs).
B. Corticosteroids.
C. Arrange for cath lab.
D. Heparin.
E. Thrombolysis.

A

A. Non-steroidal inflammatory drugs (NSAIDs).

The clinical picture and the ECG findings are characteristic of pericarditis as the most likely diagnosis. Pericarditis presents with pleuritic chest pain (worsening with respiration) that often intensifies on lying down and relieved by bending forward. A friction rub (not a given finding in this patient) may be present. Fever, often low-grade, can be a feature.

Pericarditis is an important differential diagnosis to consider in patients with chest pain. It is diagnosed in 5% of patients presenting to emergency departments with chest pain in the absence of a myocardial infarction. Viral infections and idiopathic pericarditis are the most common etiologies.

Troponin can be elevated in some patients; however, unlike in myocardial infarction, raised troponin levels in pericarditis are not associated with negative prognosis.

Pericardial effusion is common complication that can follow in as many as 60% of patients. Cardiac tamponade can be a serious life- threatening condition complicating pericardial effusion.

Findings in pericarditis can include diffuse PR depression and diffuse ST elevation with upward concavity. Unlike pericarditis, myocardial infarction typically produces ST elevation with upward convexity. ST elevation in pericarditis is often seen in pericardial and limb leads and involves more than one coronary vascular territory.

Nonsteroidal anti-inflammatory drugs (NSAIDs) are the first-line medications for treatment of pericarditis. Of NSAIDs, ibuprofen is the drug of choice due to its favorable effect on the coronary flow and large dose range. Depending on severity and response, 300–800 mg every 6–8 hours may be initially required and can be continued for days or weeks, ideally until the effusion has disappeared. Gastrointestinal protection may be considered.

Colchicine (0.5 mg twice daily) added to an NSAID or as monotherapy also appears to be effective for the initial attack and the prevention.

The role of systemic corticosteroids (option B) in pericarditis is controversial. High dose corticosteroids (i.e., prednisolone 1 mg/kg/ day) with a 2–4-week taper is considered in pericarditis secondary to connective tissue disease, uremia or autoreactivity.

Referring the patient to cath lab (option C), heparin (option D), or thrombolytic agents (option E) may be considered for patients with myocardial infarction or other conditions within the spectrum of acute coronary syndrome. The pleuritic nature of the chest pain, presence of fever, and the characteristic ECG findings favors pericarditis with high index of certainty.

89
Q

A 71-year-old man is brought the Emergency Department with right-sided weakness of his arm and leg. His past medical history is significant for myocardial infarction 3 years ago for which he takes aspirin 100 mg daily. Non-contrast brain CT scan excludes intracranial hemorrhage. Doppler ultrasonography shows bilateral carotid artery stenosis of 50%. Which one of the following would be the most appropriate initial management?

A. Add warfarin.
B. Add clopidogrel.
C. Carotid endarterectomy.
D. Continue the same dose of aspirin.
E. Increase the dose of aspirin.

A

B. Add clopidogrel

Immediate Steps After Stroke or TIA:
1. Exclude Intracranial Hemorrhage: Use a non-contrast brain CT scan.
2. Antiplatelet Therapy:
- Start aspirin plus clopidogrel within 24 hours.
- Continue dual therapy short-term.
- Note: Dual therapy is not recommended for long-term stroke/TIA prevention.

Patient Scenario:
- Patient on aspirin develops a stroke.
- Immediate Action: Add clopidogrel short-term.

Carotid Artery Stenosis Management:
- Asymptomatic Patients: Carotid artery stenosis ≥ 60% should undergo carotid endarterectomy.
- Symptomatic Patients: Stenosis ≥ 50% requires carotid endarterectomy, ideally within 2 weeks, to prevent further strokes.
- Immediate Focus: Manage current stroke/TIA, not the long-term prevention.

Option Evaluations:
- Option A: Warfarin is for preventing stroke in atrial fibrillation, not suitable here.
- Option D: Continuing the same dose of aspirin alone won’t prevent further strokes.
- Option E: Increasing aspirin dose isn’t proven to be more effective for stroke/TIA prevention.

Key Points:
- Exclude hemorrhage first.
- Use dual antiplatelet therapy short-term.
- Consider carotid endarterectomy for stenosis, but focus immediate actions on current stroke/TIA.

Current guidelines recommend immediate antiplatelet therapy for patients with stroke or TIA after intracranial hemorrhage is excluded using a non-contrast brain CT scan. Aspirin plus clopidogrel should be commenced within 24 hours and continued short-term. There is strong recommendation against dual therapy with aspirin and clopidogrel for long-term prevention of stroke/ TIA. This patient has developed a stroke while on aspirin. For him, short-term addition of clopidogrel is the most appropriate immediate step to consider.

It is recommended that asymptomatic patients with carotid artery stenosis of equal to greater than 60% and symptomatic patients with stenosis of equal to or greater than 50% undergo carotid endarterectomy in an attempt to prevent further strokes. With a cerebrovascular event and a stenosis of 50% this patent requires carotid endarterectomy, ideally within 2 weeks, as the most appropriate action to prevent further strokes. The question however asks about immediate rather than long-term management.

OPTION A : Warfarin is the preferred anticoagulant to prevent stroke in the setting of atrial fibrillation.

OPTION D : With the stenotic carotid artery, continuing the same dose of aspirin is necessary but will not prevent further cerebrovascular events.

OPTION E : Increasing the dose of aspirin has not proven effective in prevention of further strokes/TIAs compared to lower doses.

90
Q

A 56-year-old man presents to the emergency department with complaints of chest pain and lightheadedness. On examination he has a blood pressure of 85/50mmHg, rapid and barely perceptible pulse of 160bpm and respiratory rate of 24 breaths per minute. An ECG is obtained which is shown in the accompanying photograph. Which one of the following is the most appropriate next step in management?

A. Atropine.
B. Intravenous diltiazem.
C. Intravenous metoprolol.
D. Pace maker insertion.
E. DC cardioversion.

A

E. DC cardioversion.

The ECG is characteristic of supraventricular tachycardia (tachycardia with regular and monomorphic QRS complexes and absence of P wave).

SVT is a common condition that occurs in individuals of all age groups and presents with a variety of clinical manifestations. Patients may be asymptomatic or may present with minor palpitations or more severe symptoms.

If symptomatic, symptoms can include:
Palpitations - >96%
Dizziness - 75%
Shortness of breath - 47%
Syncope - 20%
Chest pain - 35%
Fatigue - 23%
Diaphoresis - 17%
Nausea - 13%

Rare complications of paroxysmal SVT include myocardial infarction, congestive heart failure, syncope, and sudden death.

Acute management of supraventricular tachycardia (SVT) includes controlling the rate and preventing hemodynamic collapse. Patients with unstable hemodynamic status (hypotension, chest pain, dyspnea and perfusion-related confusion) should be treated with immediate cardioversion after sedation. This patient has a systolic blood pressure of less than 90 mmHg indicating hemodynamic instability. Lightheadedness per se is not a sign of instability.

If the patient is stable, vagal maneuvers can be used to slow the heart rate and to convert it sinus rhythm. If vagal maneuvers are not successful, adenosine can be used in increasing doses. If adenosine does not work, atrioventricular (AV) nodal blocking agents such as calcium channel blockers (diltiazem or verapamil) or beta-blockers (e.g. metoprolol) should be used, as most patients who present with SVT have AV nodal reentrant tachycardia (AVNRT) or AV reentrant tachycardia (AVRT). These arrhythmias depend on AV nodal conduction and therefore can be terminated by transiently blocking this conduction.

Option A and D: Atropine and pace maker are used for patients with symptomatic bradycardia which is not the case here.

91
Q

A 67-year-old man, who is a known case of congestive heart failure and well controlled on treatment, forgets to take his medication while on a 10-day trip to France. Today he is in your office with complaint of bilateral edema of his legs. On examination, he has a blood pressure of 165/110mmHg, heart rate of 106bpm and respiratory rate of 23 breaths per minute. There is bilateral pitting edema of both legs up to the knees. Which one of the following options is the drug to consider for him first?

A. Spironololoctone.
B. Furosemide.
C. Digoxin.
D. Beta blockers.
E. ACE inhihitors and beta blokcers but not diuretics.

A

B. Furosemide.

Patient Presentation: A patient with previously well-controlled heart failure now presents with decompensation.

Initial Consideration: Assess volume status.

Hypervolemic Patients:
- First Step: Volume reduction using a loop diuretic (e.g., furosemide, bumetanide, torsemide).
- Start an ACE Inhibitor: (e.g., captopril, enalapril) or an ARB (e.g., losartan, irbesartan, valsartan) if intolerant to ACE inhibitors.

Indicators of Hypervolemia:
- Increased jugular venous pressure
- Weight gain
- Dyspnea
- Peripheral edema
- Lung crackles
- Elevated brain natriuretic peptide (BNP)
- Severe cases: hepatomegaly, ascites, anasarca

Next Steps Once Euvolemic:
- Add a cardioselective beta blocker to the regimen.

Euvolemic Patients:
- Manage with an ACE inhibitor (or ARB) plus a beta blocker.

Note: Beta blockers should only be started if there are no respiratory crackles and minimal peripheral edema.

Current Patient: With significant bilateral edema, indicating hypervolemia:
- Most Appropriate Next Step: Start on a loop diuretic (e.g., furosemide).
- In Conjunction: Use an ACE inhibitor or ARB for mortality benefit.

Option Evaluations:
- Option A: Spironolactone (an aldosterone antagonist) is for euvolemic patients with a GFR > 30 ml/min and serum potassium < 5 mmol/L. This patient is hypervolemic and not a candidate for spironolactone now.
- Option C: Digoxin is for patients with EF < 35%. It should be added only after achieving the maximally-tolerated dose of a beta blocker.
- Options D and E: Beta blockers are contraindicated in volume-overloaded CHF patients. They are beneficial in euvolemic CHF patients.

Summary:
For a hypervolemic patient with decompensated heart failure, the primary treatment is a loop diuretic combined with an ACE inhibitor or ARB. Beta blockers and other medications are added after achieving euvolemia.

This patient has presented with a decompensated heart failure which has been previousuly well controlled. In approaching such patients, the first thing to consider is the volume status. For patients who are in hypervolemic state, the fisrt step is volume reduction using a loop diuretic such as furosemide, bumetanide, or torsemide. It is also important to start the patient on an ACE inhibitor (e.g. captopril, enalapril). Angiotensin receptor blockers (ARBs) such as losartan, erbisartan or valsartan can be used alternatively in those who are intolerant of ACE inhibitors.

Indicators of hypervolemic state neccessiating commencement of a diuretic are:
* Increased jugular venous pressure
* Increased weight
* Dyspnea
* More than minimal peripheral edema
* Crackels
* Elevated brain natriuretic peptide
* Hepatomegaly, ascites or anasarca in severe cases

Once the patient is euvolemic, addition of a cardioselective beta blocker to the above regimen is the next best step in management.

For euvolemic patients, an ACE inhibitor (or ARB) plus a beta blocker is the management of choice.

NOTE - beta blokers should only be started for patients who do not have respiratory crackles and no more than minimal peripheral edema.

With significant bilateral edema in this patient, he is hypervolemic and should be started on a loop diuretic such as furosemide as the most appropriate next step in management. An ACE inhibitor or ARB should be used in conjunction with a diuretic for mortality benefit.

Option A: Spironolactone (an aldosterone antagonist) is used in euvolemic patients who have a GFR>30ml/min and serum potassium of < 5mmol/L with close monitoring of potassium levels. Spironolactone provides additional mortality benefit in patients with CHF. Since this patient is hypervolemic, he cannot be started on spironolactone now.

Option C: Digoxin is indicated in patients with EF < 35%. Digoxin should be added only after maximally-tolerated dose of a beta blocker.

Option D and E: Beta blockers are contraindicated in patients with CHF who have volume overload evident by significant peripheral edema, pulmonary crackles, increased JVP, ascites, hepatomegaly, etc. In euvolemic patients, however, addition of beta blockers has shown mortality benefit. This drug class should be considered for all patients with euvolemic CHF.

92
Q

A 75-year-old man comes to the Emergency Department because of severe chest pain starting 20 minutes ago. His past medical history includes type II diabetes mellitus and hypertension. On examination, he has a blood pressure of 110/65 mmHg and pulse of 110 bpm. An ECG shows ST elevation in leads I, aVL, V5 and V6. With the diagnosis of ST elevation myocardial infarction, he undergoes thrombolytic therapy with tenecteplase followed by heparin infusion. ST segment elevation resolves within 20 minutes as does the chest pain. While heparin is being infused, he suddenly develops drowsiness and confusion. His blood pressure is now 185/110 mmHg and pulse rate 50 bpm. There is no focal neurological finding. Which one of the following is the best explanation for this clinical picture?

A. Intracranial hemorrhage.
B. Silent myocardial re-infarction.
C. Anaphylactic reaction to tenecteplase.
D. Hypertensive encephalopathy.
E. Aortic dissection.

A

A. Intracranial hemorrhage.

A. Intracranial Hemorrhage

Key Points:
- Symptoms: Sudden lethargy, confusion, high blood pressure, and low heart rate.
- Cause: Increased intracranial pressure from bleeding in the brain.
- Risk with Thrombolytic Therapy: Occurs in 1% of patients, with high mortality (53-60%) and severe outcomes (25% have permanent deficits).

Why Not the Others?

B. Silent Myocardial Reinfarction:
- Presents with bradycardia and low blood pressure (if inferior reinfarct).
- Not associated with high blood pressure and bradycardia.

C. Anaphylactic Reaction to Tenecteplase:
- Causes low blood pressure, not high blood pressure.

D. Hypertensive Encephalopathy:
- Develops slowly and requires very high blood pressure (>180/120 mmHg).
- Symptoms include headache, restlessness, vomiting, confusion, and coma.

E. Aortic Dissection:
- Would cause compensatory tachycardia (fast heart rate), not bradycardia.

Sudden onset of lethargy and confusion, as well as raised blood pressure and bradycardia are consistent with the diagnosis of elevated intracranial pressure due to intracranial bleeding.

Intracranial hemorrhage is a devastating complication of thrombolytic therapy seen in 1% of patients. 53-60% of patients die of the event during hospitalization and 25% of the survivors will suffer permanent neurological deficits.

Option B: Silent myocardial reinfarction may present with bradycardia and hypotension if an inferior reinfarct occurred, but not with hypertension and bradycardia.

Option C: Anaphylactic reaction to tenecteplase would have caused hypotension, not hypertension.

Option D: Hypertensive encephalopathy has a more insidious onset and is seen in higher blood pressures (>180/120mmHg). It presents with headache, restlessness, vomiting, confusion and coma.

Option E: With aortic dissection there should have been compensatory tachycardia.

93
Q

A 50-year-old man presents to your practice for treatment of his newly-diagnoses hypertension. His medical history is significant for asthma and reflux nephropathy. Laboratory studies shows a serum urea of 18 mmol/L (normal: 2.9-8.2 mmol/L), creatinine of 190 μmol/L (normal: 50- 110 μmol/L) and proteinuria of 900 mg/d. Which one of the following would be the antihypertensive medication of choice for him?

A. Amlodipine.
B. Losartan.
C. Perindopril.
D. Indapamide.
E. Atenolol.

A

A. Amlodipine.

Selecting Antihypertensive Medications: Key Considerations

Main Factors:
- Concomitant medical conditions
- Contraindications to specific drug classes

  1. Migraine and Hypertension:
    • Best Option: Beta blockers
    • Reason: Lowers blood pressure and prevents migraines.
  2. Proteinuria and Hypertension:
    • Best Options: ACE inhibitors or ARBs (e.g., perindopril, losartan)
    • Exception: Not suitable for patients with significantly decreased GFR (e.g., polycystic kidney disease, high BUN and creatinine).
      • Reason: These drugs relax the efferent glomerular arteriole, worsening the already impaired GFR.
  3. Chronic Kidney Disease (CKD) with Reduced GFR:
    • Avoid: ACE inhibitors or ARBs
      • Reason: Reduces intra-glomerular pressure and GFR.
    • Best Options: Calcium channel blockers (e.g., amlodipine) or beta blockers (e.g., atenolol, metoprolol)
      • Exception: Avoid beta blockers if the patient has asthma (beta blockers are contraindicated).
  4. Renal Disease with Reduced GFR:
    • Avoid: Diuretics (e.g., indapamide)
      • Reason: Less effective in renal disease.

Patient with Hypertension, Proteinuria, and Polycystic Kidney Disease with Decreased GFR, and Asthma:

  • Best Option: Calcium channel blocker (e.g., amlodipine)
  • Reason: Effective for hypertension, safe for reduced GFR, and no asthma contraindication.

Summary:
- Calcium Channel Blockers: Good for patients with kidney disease and decreased GFR.
- Beta Blockers: Good for certain conditions but avoid if the patient has asthma.
- ACE Inhibitors/ARBs: Avoid if the patient has significantly decreased GFR.
- Diuretics: Less reliable in renal disease.

Important determining factors for selection of antihypertensive medications include the presence of concomitant medical conditions and/or contraindications to a specific antihypertensive drug class. For example, patients with migraine and hypertension may benefit from beta blockers because beta blockers can simultaneously lower the blood pressure and prevent migraine attacks. Patients with proteinuria and hypertension are likely to benefit most from ACE inhibitors or angiotensin receptor blockers (ARBs), while these agents are not appropriate management options for treatment of patients with chronic kidney disease associated with significantly decreased GFR, because these drugs act by relaxing of the efferent glomerular arteriole that will result in deterioration of an already impaired GFR.

Although the proteinuria makes ACE inhibitors (e.g. perindopril) (option C) or ARBs (e.g. losartan) (option B) the most appropriate option for him, the polycystic kidney disease associated with significantly decreased GFR (evident by increased BUN and creatinine) makes them inappropriate options as well (for calculation of eGFR based on the serum creatinine, age and sex click here). In patients with bilateral renal artery stenosis, hypertensive nephrosclerosis, polycystic kidney disease or chronic kidney disease, intra-renal perfusion pressure is already reduced, and GFR is maintained in part by an angiotensin II-induced increase in resistance of the efferent (post-glomerular) arteriole. Blocking this response with an ACE inhibitor or ARB will relax the efferent arteriole, lower intra-glomerular pressure, and reduce the glomerular filtration rate.

Indapamide (option D) is a diuretic. The antihypertensive effect of diuretics is reduced in patients with renal disease making them inappropriate and less reliable options for hypertension control in such patients.

In patients with renal disease associated with decreased GFR, calcium channel blocker such as amlodipine and nifedipine and beta blockers such as atenolol and metoprolol are acceptable options; however, this patient has asthma that makes beta blockers contraindicated for him. Of the options, the only appropriate medication to consider for hypertension control in this man is the calcium channel blocker amilodipine.

94
Q

Robert, 67 years old, presents to your clinic for evaluation of increasing left leg pain for the past 6 months. He describes the pain as aching which was brought on after walking one or two blocks and alleviated with rest. However, it has been worse recently, starting after walking shorter distances and taking more time to get better. He also mentions that sometimes he wakes up in the middle of the night due to leg pain. He smokes 25-30 cigarettes a day and drinks socially. He denies any remarkable medical condition except hypercholesterolemia for which he is on atorvastatin 10 mg, daily. On examination, he has a blood pressure of 142/91mmHg, pulse rate of 88 bpm and a BMI of 32. Compared to the right side, left leg has less hair. He has a left ankle/brachial index (ABI) of 0.35 on the left side and 0.7 on the right. Which one of the following investigations is the option of choice to establish a certain diagnosis in Robert?

A. Duplex ultrasound.
B. Digital subtraction catheter (DSA) angiography.
C. Arteriography.
D. CT angiography.
E. Magnetic resonance angiography.

A

A. Duplex ultrasound.

Robert has typical features of ischemic leg pain in history and physical examination. An ABI of 0.3 (normal: 0.9-1.1) signifies the need for endovascular intervention. However, confirmatory investigations are still required to establish and document such diagnosis.

To make a diagnosis of arterial disease noninvasively, ultrasound examination is the mainstay investigation and the best option here. Each mode of ultrasound provides specific information. Ultrasonography is used to evaluate the presence of vascular disease, anatomic location, and the extent and morphology of the lesion(s). Duplex ultrasound scan is the most appropriate diagnostic tool to use for Robert to establish a diagnosis of vascular disease.

Real-time ultrasound scan uses reflected sound waves to produce images and assess blood velocity. Two ultrasound modes are routinely used in vascular imaging: (1) B Mode and (2) doppler mode.

B mode (brightness mode) is the conventional, grayscale, or 2-dimensional ultrasound scan. This mode is used for anatomical details. Color doppler or simply doppler ultrasound scan is the mode used for assessment of blood flow and velocity. The combination of these two (B mode + doppler mode) is called duplex ultrasound.

There also other advanced imaging modalities such as CT angiography (option D) or Magnetic resonance angiography (MRA) (option E) . The main goal of these modalities is provision of information necessary for intervention either by open surgery or endovascular intervention. These modalities are not used for diagnosing vascular disease because the non-invasive and readily available ultrasound can diagnose vascular disease in most anatomical sites with high sensitivity and specificity.

Catheter DSA (option B) is the gold standard for imaging peripheral arteries but is rarely used for diagnosis because of its invasive nature and the availability of non-invasive or less invasive imaging modalities such as duplex ultrasound, CT angiography and MRA.

DSA is used to clearly visualize blood vessels in a bony or dense soft tissue environment. A catheter is introduced into an artery (e.g. tibial artery) and contrast medium is injected into that artery. Images are produced by subtracting a pre-contrast image from post-contrast images.

Option E: Arteriography is another term used for angiography. In general, conventional angiography or arteriography consists of injection of contrast matter into arteries and taking X-ray images. As mentioned before, these techniques are used mainly (if ever these days) before interventions. For diagnosis, ultrasound is the best option.

NOTE - In cases with traumatic limb ischemia or acute limb-threatening ischemia where endovascular or open surgery is required, CT angiography is the modality of choice because urgent intervention is considered and should be planned. MRA is another option but it is less available. In such cases, duplex ultrasound is used only if CT angiography or MRI are not available or the patient has any contraindications for contrast matter. However, some surgeons still prefer the duplex as the modality of choice to guide the operation.

Sure, here’s a simplified summary:

Diagnosing Ischemic Leg Pain: Best Imaging Options

  1. Duplex Ultrasound:
    • Uses: Checks for vascular disease and blood flow.
    • Advantages: Non-invasive, accurate, widely available.
  2. CT Angiography (CTA):
    • Uses: Detailed images with contrast.
    • When: Urgent cases needing quick intervention planning.
  3. MRI Angiography (MRA):
    • Uses: Magnetic fields and contrast.
    • When: Alternative to CTA, especially if CT contrast is a concern.
  4. DSA (Digital Subtraction Angiography):
    • Uses: Invasive, precise images with contrast.
    • When: Before specific interventions, less commonly for initial diagnosis.

Remember: Start with ultrasound for most cases of leg pain. Use CTA or MRA in emergencies or if ultrasound isn’t possible. DSA is for detailed planning before procedures.

95
Q

A 65-year-old man presents to the Emergency Department with complaint of chest pain radiating to his jaw for the past 2 hours. The patient is given aspirin, sublingual nitroglycerine (TNG), and supplemental oxygen. With these measures the chest pain subsides. An ECG is obtained and is as follows. A troponin level is requested with the result still pending. Which one of the following is the next best step in management?

A. Send the patient home on aspirin.
B. Coronary angiogram and angioplasty.
C. Thrombolysis.
D. Admit and wait for the troponin results.
E. Perform stress test.

A

B. Coronary angiogram and angioplasty.

This patient as a typical ischemic chest pain as well as an ECG showing ST segment elevation in leads II, III, and aVF. With these, an inferior ST segment elevation myocardial infarction (STEMI) is a definite diagnosis prompting immediate treatment and reperfusion therapy.

Therapy should be started regardless of the troponin result because troponin may take up to 6-8 hours to become positive, and its negativity does not change the diagnosis and the management plan in this scenario. In other words, even with a negative troponin result, the diagnosis is still STEMI and urgent reperfusion therapy is required.

According to National Heart Foundation of Australia, Australian Clinical Guidelines for the Management of Acute Coronary Syndromes For patients with ST elevation myocardial infarction (STEMI) presenting within 12 hours of symptom onset, and in the absence of comorbidities that impact the individual’s overall survival, emergency reperfusion therapy with either primary percutaneous coronary intervention (PCI) (formerly called angioplasty) or fibrinolytic therapy is recommended.

Since this patient is still in the 12 hours window, angiography to located the coronary stenosis or blockage and stent placement to maintain the vessel patent is the first-line treatment choice. In case PCI cannot be performed with acceptable delay, thrombolysis using agents such as alteplase, tenecteplase or reteplase are the treatment of choice.

96
Q

A 62-year-old man presents to your clinic for a follow-up visit. He had a myocardial infarction (MI) 4 years ago. He is currently on aspirin 80 mg daily. He follows a healthy diet and does exercise regularly. Laboratory studies are all within normal parameters. An ECG is obtained that is normal. Which one of the following is the most appropriate advice for him?

A. Reinforce the importance of a healthy diet and regular exercise.
B. Stop aspirin.
C. Add statins.
D. Tell him he does not need any further follow-up.
E. Add warfarin.

A

C. Add statins.

According to the guideline by National Heart Foundation of Australia, every post-MI patient have to should be provided with the following medications and advice upon discharge. Of these, some are indicated in all post-MI patients and some are only used under certain circumstances (read explanations for each entity):

  • Aspirin: All patients should take 75–150 mg daily unless contraindicated.
  • Clopidogrel: There is evidence that clopidogrel should be prescribed for up to 12 months after an acute coronary syndrome, in particular after stent implantation, with the duration of therapy depending on the particular type of stent and circumstances of implantation (level II evidence). Clopidogrel may also be prescribed as an alternative when aspirin is contraindicated, or in addition to aspirin, particularly in patients with unstable angina or recurrent cardiac events.
  • Beta-blocker: Should be prescribed for most patients after a myocardial infarction unless contraindicated, and continued indefinitely, especially in high-risk patients. Carvedilol, bisoprolol or metoprolol (extended release) should be used in patients with heart failure.
  • Angiotensin-converting enzyme inhibitor: Should be given early after an acute coronary syndrome, and its use reviewed later.
  • Statins: Statin therapy should be initiated in hospital for all patients with coronary heart disease.
  • Warfarin: Recommended after myocardial infarction for those at high risk of systemic thromboembolism because of atrial fibrillation, mural thrombus, congestive heart failure or previous embolization. Warfarin may sometimes be combined with aspirin, but in this circumstance patients should be observed closely for signs of bleeding.
  • Nitrates: All patients should be prescribed a short-acting nitrate (unless contraindicated) and provided with a written action plan for chest pain.
  • Insulin/oral hypoglycemics: Good glycaemic control should be obtained and continued in patients who have had acute coronary syndromes and who have diabetes.
  • Aldosterone antagonists: Initiation of eplerenone therapy should be considered early after myocardial infarction in those with left-ventricular systolic dysfunction and symptoms of heart failure.
  • Lifestyle advice: All patients should be given advice on lifestyle changes that will reduce the risk of further cardiac events, including smoking cessation, good nutrition, moderate alcohol intake, regular physical activity and weight management as appropriate.
  • Ongoing prevention and cardiac rehabilitation programs: Cardiac rehabilitation is a proven effective intervention. All patients with cardiovascular disease should have access, and be actively referred, to comprehensive ongoing prevention and cardiac rehabilitation services.
  • Chest pain action plan
    All patients should be provided with a written action plan for chest pain which includes:
    1. Rest and self-administration of short-acting nitrates
    2. Self-administration of aspirin unless contraindicated
    3. Calling an ambulance if chest pain or discomfort is not completely relieved within 10 minutes
    4. Individualised clinician notification and action plan for those living in areas where an ambulance is not readily available.
  • Fish oil: A diet high in omega-3 fatty acids from fish and the use of fish oil tablets is recommended (grade B recommendation).
  • Psychosocial factors: Depression, social isolation and lack of quality social support are likely to lead to significantly worse outcomes in those with coronary heart disease. All patients with coronary heart disease should be assessed for comorbid depression and level of social support.
  • Diabetes: An early glucose tolerance test should be considered in those without diagnosed diabetes.
  • Implantable cardiac defibrillators (ICDs): ICDs should be considered in some patients who, despite optimal medical therapy, have persistently depressed left ventricular function after ST-segment-elevation myocardial infarction.

This patient is already taking aspirin and should continue it for life unless contraindicated. Therefore, cessation of aspirin (option B) would be inappropriate. He is already following a healthy life style including diet and exercise. It is a good idea to encourage him and reinforce the importance of a healthy diet and regular exercise (option A). He should have been discharged on statins. In fact, statins must have been started during his hospital stay and continued indefinitely unless contraindicated. Of the options, adding statins to his medications is the** first priority** and should come first. The next best advice would be reinforcing the importance of a healthy diet and regular exercise.

Option D: This patient requires regular follow-ups and monitoring. Telling the patient that he does need follow up is an incorrect advice.

Option E: Warfarin is only recommended after myocardial infarction for those at high risk of systemic thromboembolism due to atrial fibrillation, mural thrombus, congestive heart failure, or previous embolization. This is not indicated in this patient.

Certainly! Here’s a simplified breakdown of the key points for post-MI patient management according to the National Heart Foundation of Australia:

  1. Aspirin: Generally recommended unless contraindicated.
  2. Clopidogrel: Used for up to 12 months post-acute coronary syndrome or as an alternative to aspirin in certain cases.
  3. Beta-blocker: Prescribed unless contraindicated, especially in high-risk patients.
  4. ACE Inhibitor: Initiated early after MI, reviewed later.
  5. Statins: Started in hospital and continued indefinitely.
  6. Nitrates: Provided for chest pain, unless contraindicated.
  7. Insulin/Oral Hypoglycemics: Ensure good glycemic control in diabetic patients.
  8. Aldosterone Antagonists: Considered in heart failure with reduced ejection fraction.
  9. Lifestyle Advice: Includes smoking cessation, healthy diet, moderate alcohol, regular exercise, and weight management.
  10. Cardiac Rehabilitation: Recommended for all patients.
  11. Chest Pain Action Plan: Includes rest, nitroglycerin, aspirin, and when to seek medical help.
  12. Fish Oil: Omega-3 fatty acids recommended.
  13. Psychosocial Factors: Assess for depression and social support.
  14. Diabetes: Consider glucose tolerance testing.
  15. ICDs: Considered in patients with persistently reduced heart function post-MI.

Remember: Start statins during hospital stay, continue for life unless contraindicated. Reinforce lifestyle changes and provide a chest pain action plan for all patients. Regular follow-up is essential, and warfarin is only for specific high-risk cases.

97
Q

A 32-year-old woman presents to you practice with complaint of an upper respiratory tract infection evident by a fever of 38.3°, rhinorrhea, and dry cough. On examination, her blood pressure is 170/110 mmHg on two separate readings 15 minutes apart. On auscultation, an abdominal bruit is heard. The bruit radiates to the right flank. A Doppler duplex sonography shows right renal artery stenosis. Which one of the following would be the most appropriate next step in management?

A. Conventional renal arteriography.
B. Magnetic resonance angiography (MRA).
C. Start her on enalapril.
D. Start her on thiazide diuretics.
E. Perform percutaneous renal artery angioplasty.

A

C. Start her on enalapril.

This woman has been incidentally found to have unilateral renal artery stenosis (RAS) once the cause of her hypertension has been investigated.

Renal artery stenosis (RAS) is the major cause of renovascular hypertension. Apart from its role in the pathogenesis of hypertension, RAS is also being increasingly recognized as an important cause of chronic renal insufficiency and end-stage renal disease. In older individuals, atherosclerosis is by far the most common etiology of RAS. As the renal artery lumen progressively narrows, renal blood flow decreases. Eventually, the decreased perfusion compromises renal function and structure. Less commonly, RAS may have been caused by fibromascular dysplasia. This more often is seen in younger patients.

RAS may present with one or more of the following:
* Abdominal bruit
* Azotemia
* Sudden worsening of hypertension or renal function
* Acute kidney injury or decreased renal function after initiation of antihypertensive therapy, especially with angiotensin-converting enzyme (ACE) inhibitors or angiotensin receptor blockers
* Unexplained renal insufficiency, in elderly patients.
* Congestive heart failure, with poor control of hypertension and renal insufficiency in the absence of a significant decrease in ejection fraction (so-called flash pulmonary edema)

Investigations to consider in patients with suspected RAS include laboratory and imaging studies. Laboratory studies include renal function test such as creatinine, urea, and electrolytes, 24-hour urine exam, and urinalysis. Serologic tests may also be considered for systemic lupus erythematosus or vasculitis if these conditions are suspected.

The initial imaging to consider for patients with suspected RAS is Duplex ultrasound scanning which is a combination of B-mode (conventional) ultrasound for assessment of anatomical structures and Doppler unit for obtaining data about renal blood flow velocity. This technique is non-invasive and has a high sensitivity and specificity (~98%).

The general approach to treatment of RAS includes control of hypertension and hypercholesterolemia in patients with RAS due to atherosclerosis. Preferred antihypertensive medications in RAS are angiotensin-converting enzyme (ACE) inhibitors or angiotensin receptor blockers (ARBs). These two classes of medications can lead to impaired renal function evident by increased serum creatinine and hyperkalemia. It is recommended that such patients should be monitored closely, and if creatinine increases by >25% of baseline within the first month of use, these drugs are ceased and replaced with calcium channel blockers as the second-line medications for hypertension control. Diuretics including thiazide diuretics (option D) are not recommended for hypertension control in patients with RAS.

Also, strict serum cholesterol control is recommended for patients with RAS in the setting of atherosclerosis usually with statins.

In the absence of trials showing benefit from revascularisation therapy such as percutaneous renal artery angioplasty (option E) over conventional non-surgical treatment and the significant risk of complications, revascularization treatments is only considered and advised for patients who fail medical therapy with resistant or poorly-controlled hypertension; recurrent flash pulmonary edema; dialysis-dependent kidney failure resulting from renal artery stenosis; chronic renal insufficiency and bilateral renal artery stenosis; or renal artery stenosis of a solitary functioning kidney. This patient may or may not need revascularization after further assessment is undertaken. In the meanwhile, she should be started on enalapril (ACE inhibitor) for hypertension control as the most appropriate next step in management.

Other imaging studies such as conventional renal arteriography (option A), spiral CT angiography, MR angiography (option B), or radionuclide scans may be considered for further assessment and treatment. None of such investigations take precedence over starting therapy with antihypertensive medications as this stage.